SAUNDERS HESI MED SURG

¡Supera tus tareas y exámenes ahora con Quizwiz!

The nurse is applying a topical corticosteroid to a client with eczema. The nurse should apply the medication to which body area? (SELECT ALL THAT APPLY.) 1. Back 2. Axilla 3. Eyelids 4. Soles of the feet 5. Palms of the hands

1, 4, 5 Topical corticosteroids can be absorbed into the systemic circulation. Absorption is higher from regions where the skin is especially permeable (scalp, axilla, face, eyelids, neck, perineum, genitalia), and lower from regions where permeability is poor (back, palms, soles). The nurse should avoid areas of higher absorption to prevent systemic absorption.

The nurse is caring for a client with a diagnosis of influenza who first began to experience symptoms yesterday. Antiviral therapy is prescribed and the nurse provides instructions to the client about the therapy. Which statement by the client indicates an understanding of the instructions? 1. "I must take the medication exactly as prescribed." 2. "Once I start the medication, I will no longer be contagious." 3. "I will not get any colds or infections while taking this medication." 4. "This medication has minimal side effects and I can return to normal activities."

1 Antiviral medications for influenza must be taken exactly as prescribed. These medications do not prevent the spread of influenza and clients are usually contagious for up to 2 days after the initiation of antiviral medications. Secondary bacterial infections may occur despite antiviral treatment. Side effects occur with these medications and may necessitate a change in activities, especially when driving or operating machinery if dizziness occurs.

A client had a new colostomy created 2 days earlier and is beginning to pass malodorous flatus from the stoma. What is the correct interpretation by the nurse? 1. This is a normal, expected event. 2. The client is experiencing early signs of ischemic bowel. 3. The client should not have the nasogastric tube removed. 4. This indicates inadequate preoperative bowel preparation.

1 As peristalsis returns following creation of a colostomy, the client begins to pass malodorous flatus. This indicates returning bowel function and is an expected event. Within 72 hours of surgery, the client should begin passing stool via the colostomy. Options 2, 3, and 4 are incorrect interpretations.

The nurse is reviewing the history and physical examination of a client who will be receiving asparaginase, an antineoplastic agent. The nurse contacts the health care provider before administering the medication if which disorder is documented in the client's history? 1. Pancreatitis 2. Diabetes mellitus 3. Myocardial infarction 4. Chronic obstructive pulmonary disease

1 Asparaginase is contraindicated if hypersensitivity exists, in pancreatitis, or if the client has a history of pancreatitis. The medication impairs pancreatic function and pancreatic function tests should be performed before therapy begins and when a week or more has elapsed between dose administrations. The client needs to be monitored for signs of pancreatitis, which include nausea, vomiting, and abdominal pain. The conditions noted in options 2, 3, and 4 are not contraindicated with this medication.

The nurse is teaching the client about his prescribed prednisone. Which statement, if made by the client, indicates that further teaching is necessary? 1. "I can take aspirin or my antihistamine if I need it." 2. "I need to take the medication every day at the same time." 3. "I need to avoid coffee, tea, cola, and chocolate in my diet." 4. "If I gain more than 5 pounds (2.25 kg) a week, I will call my health care provider (HCP)."

1 Aspirin and other over-the-counter medications should not be taken unless the client consults with the HCP. The client needs to take the medication at the same time every day and should be instructed not to stop the medication. A slight weight gain as a result of an improved appetite is expected; however, after the dosage is stabilized, a weight gain of 5 pounds (2.25 kg) or more weekly should be reported to the HCP. Caffeine-containing foods and fluids need to be avoided because they may contribute to steroid-ulcer development.

The clinic nurse is performing an admission assessment on a client and notes that the client is taking azelaic acid. The nurse determines that which client complaint may be associated with use of this medication? 1. Itching 2. Euphoria 3. Drowsiness 4. Frequent urination

1 Azelaic acid is a topical medication used to treat mild to moderate acne. Adverse effects include burning, itching, stinging, redness of the skin, and hypopigmentation of the skin in clients with a dark complexion. The effects noted in the other options are not specifically associated with this medication.

The nurse is monitoring a client newly diagnosed with diabetes mellitus for signs of complications. Which sign or symptom, if exhibited in the client, indicates that the client is at risk for chronic complications of diabetes if the blood glucose is not adequately managed? 1. Polyuria 2. Diaphoresis 3. Pedal edema 4. Decreased respiratory rate

1 Chronic hyperglycemia, resulting from poor glycemic control, contributes to the microvascular and macrovascular complications of diabetes mellitus. Classic symptoms of hyperglycemia include polydipsia, polyuria, and polyphagia. Diaphoresis may occur in hypoglycemia. Hypoglycemia is an acute complication of diabetes mellitus; however, it does not predispose a client to the chronic complications of diabetes mellitus. Therefore, option 2 can be eliminated because this finding is characteristic of hypoglycemia. Options 3 and 4 are not associated with diabetes mellitus.

The nurse is teaching a client about the risk factors associated with colorectal cancer. The nurse determines that further teaching is necessary related to colorectal cancer if the client identifies which item as an associated risk factor? 1. Age younger than 50 years 2. History of colorectal polyps 3. Family history of colorectal cancer 4. Chronic inflammatory bowel disease

1 Colorectal cancer risk factors include age older than 50 years, a family history of the disease, colorectal polyps, and chronic inflammatory bowel disease.

The nurse is providing discharge teaching for a client with newly diagnosed Crohn's disease about dietary measures to implement during exacerbation episodes. Which statement made by the client indicates a need for further instruction? 1. "I should increase the fiber in my diet." 2. "I will need to avoid caffeinated beverages." 3. "I'm going to learn some stress reduction techniques." 4. "I can have exacerbations and remissions with Crohn's disease."

1 Crohn's disease is an inflammatory disease that can occur anywhere in the gastrointestinal tract but most often affects the terminal ileum and leads to thickening and scarring, a narrowed lumen, fistulas, ulcerations, and abscesses. It is characterized by exacerbations and remissions. If stress increases the symptoms of the disease, the client is taught stress management techniques and may require additional counseling. The client is taught to avoid gastrointestinal stimulants containing caffeine and to follow a high-calorie and high protein diet. A low-fiber diet may be prescribed, especially during periods of exacerbation.

The nurse teaches a client about the effects of diphenhydramine, which has been prescribed as a cough suppressant. The nurse determines that the client needs further instruction if the client makes which statement? 1. "I will take the medication on an empty stomach." 2. "I won't drink alcohol while taking this medication." 3. "I won't do activities that require mental alertness while taking this medication." 4. "I will use sugarless gum, candy, or oral rinses to decrease dryness in my mouth."

1 Diphenhydramine has several uses, including as an antihistamine, antitussive, antidyskinetic, and sedativehypnotic. Instructions for use include taking with food or milk to decrease gastrointestinal upset and using oral rinses, sugarless gum, or hard candy to minimize dry mouth. Because the medication causes drowsiness, the client should avoid use of alcohol or central nervous system depressants, operating a car, or engaging in other activities requiring mental awareness during use.

The nurse is monitoring a client for the early signs and symptoms of dumping syndrome. Which findings indicate this occurrence? 1. Sweating and pallor 2. Bradycardia and indigestion 3. Double vision and chest pain 4. Abdominal cramping and pain

1 Early manifestations of dumping syndrome occur 5 to 30 minutes after eating. Symptoms include vertigo, tachycardia, syncope, sweating, pallor, palpitations, and the desire to lie down.

The nurse is preparing to care for a burn client scheduled for an escharotomy procedure being performed for a third-degree circumferential arm burn. The nurse understands that which finding is the anticipated therapeutic outcome of the escharotomy? 1. Return of distal pulses 2. Brisk bleeding from the site 3. Decreasing edema formation 4. Formation of granulation tissue

1 Escharotomies are performed to relieve the compartment syndrome that can occur when edema forms under nondistensible eschar in a circumferential third-degree burn. The escharotomy releases the tourniquet-like compression around the arm. Escharotomies are performed through avascular eschar to subcutaneous fat. Although bleeding may occur from the site, it is considered a complication rather than an anticipated therapeutic outcome. Usually, direct pressure with a bulky dressing and elevation control the bleeding, but occasionally an artery is damaged and may require ligation. Escharotomy does not affect the formation of edema. Formation of granulation tissue is not the intent of an escharotomy.

The health care provider (HCP) prescribes exenatide for a client with type 1 diabetes mellitus who takes insulin. The nurse should plan to take which most appropriate intervention? 1. Withhold the medication and call the HCP, questioning the prescription for the client. 2. Administer the medication within 60 minutes before the morning and evening meal. 3. Monitor the client for gastrointestinal side effects after administering the medication. 4. Withdraw the insulin from the prefilled pen into an insulin syringe to prepare for administration.

1 Exenatide is an incretin mimetic used for type 2 diabetes mellitus only. It is not recommended for clients taking insulin. Hence, the nurse should withhold the medication and question the HCP regarding this prescription. Although options 2 and 3 are correct statements about the medication, in this situation the medication should not be administered. The medication is packaged in prefilled pens ready for injection without the need for drawing it up into another syringe.

The nurse is assessing the colostomy of a client who has had an abdominal perineal resection for a bowel tumor. Which assessment finding indicates that the colostomy is beginning to function? 1. The passage of flatus 2. Absent bowel sounds 3. The client's ability to tolerate food 4. Bloody drainage from the colostomy

1 Following abdominal perineal resection, the nurse would expect the colostomy to begin to function within 72 hours after surgery, although it may take up to 5 days. The nurse should assess for a return of peristalsis, listen for bowel sounds, and check for the passage of flatus. Absent bowel sounds would not indicate the return of peristalsis. The client would remain NPO (nothing by mouth) until bowel sounds return and the colostomy is functioning. Bloody drainage is not expected from a colostomy

A client has developed hepatitis A after eating contaminated oysters. The nurse assesses the client for which expected assessment finding? 1. Malaise 2. Dark stools 3. Weight gain 4. Left upper quadrant discomfort

1 Hepatitis causes gastrointestinal symptoms such as anorexia, nausea, right upper quadrant discomfort, and weight loss. Fatigue and malaise are common. Stools will be light- or clay-colored if conjugated bilirubin is unable to flow out of the liver because of inflammation or obstruction of the bile ducts.

A client with hiatal hernia chronically experiences heartburn following meals. The nurse should plan to teach the client to avoid which action because it is contraindicated with a hiatal hernia? 1. Lying recumbent following meals 2. Consuming small, frequent, bland meals 3. Taking H2-receptor antagonist medication 4. Raising the head of the bed on 6-inch (15 cm) blocks

1 Hiatal hernia is caused by a protrusion of a portion of the stomach above the diaphragm where the esophagus usually is positioned. The client usually experiences pain from reflux caused by ingestion of irritating foods, lying flat following meals or at night, and eating large or fatty meals. Relief is obtained with the intake of small, frequent, and bland meals; use of H2-receptor antagonists and antacids; and elevation of the thorax following meals and during sleep.

A client with acquired immunodeficiency syndrome (AIDS) has histoplasmosis. The nurse should assess the client for which expected finding? 1. Dyspnea 2. Headache 3. Weight gain 4. Hypothermia

1 Histoplasmosis is an opportunistic fungal infection that can occur in the client with AIDS. The infection begins as a respiratory infection and can progress to disseminated infection. Typical signs and symptoms include fever, dyspnea, cough, and weight loss. Enlargement of the client's lymph nodes, liver, and spleen may occur as well.

The nurse is creating a plan of care for the client with multiple myeloma and includes which priority intervention in the plan? 1. Encouraging fluids 2. Providing frequent oral care 3. Coughing and deep breathing 4. Monitoring the red blood cell count

1 Hypercalcemia caused by bone destruction is a priority concern in the client with multiple myeloma. The nurse should administer fluids in adequate amounts to maintain a urine output of 1.5 to 2 L/day; this requires about 3 L of fluid intake per day. The fluid is needed not only to dilute the calcium overload but also to prevent protein from precipitating in the renal tubules. Options 2, 3, and 4 may be components of the plan of care but are not the priority in this client.

The nurse provides instructions to a client about measures to treat inflammatory bowel syndrome (IBS). Which statement by the client indicates a need for further teaching? 1. "I need to limit my intake of dietary fiber." 2. "I need to drink plenty, at least 8 to 10 cups daily." 3. "I need to eat regular meals and chew my food well." 4. "I will take the prescribed medications because they will regulate my bowel patterns."

1 IBS is a functional gastrointestinal disorder that causes chronic or recurrent diarrhea, constipation, and/or abdominal pain and bloating. Dietary fiber and bulk help to produce bulky, soft stools and establish regular bowel elimination habits. Therefore, the client should consume a high-fiber diet. Eating regular meals, drinking 8 to 10 cups of liquid a day, and chewing food slowly help to promote normal bowel function. Medication therapy depends on the main symptoms of IBS. Bulk-forming laxatives or antidiarrheal agents or other agents may be prescribed.

The nurse is teaching a client with hyperparathyroidism on how to manage the condition at home. Which response by the client indicates the need for additional teaching? 1. "I should limit my fluids to 1 liter per day." 2. "I should use my treadmill or go for walks daily." 3. "I should follow a moderate-calcium, high-fiber diet." 4. "My alendronate helps to keep calcium from coming out of my bones."

1 In hyperparathyroidism, clients experience excess parathyroid hormone (PTH) secretion. A role of PTH in the body is to maintain serum calcium homeostasis. When PTH levels are high, there is excess bone resorption (calcium is pulled from the bones). In clients with elevated serum calcium levels, there is a risk of nephrolithiasis. One to 2 liters of fluids daily should be encouraged to protect the kidneys and decrease the risk of nephrolithiasis. Moderate physical activity, particularly weight-bearing activity, minimizes bone resorption and helps to protect against pathological fracture. Walking, as an exercise, should be encouraged in the client with hyperparathyroidism. Clients should follow a moderate-calcium, high-fiber diet. Even though serum calcium is already high, clients should follow a moderate-calcium diet because a low-calcium diet will surge PTH. Calcium causes constipation, so a diet high in fiber is recommended. Alendronate is a bisphosphate that inhibits bone resorption. In bone resorption, bone is broken down and calcium is deposited into the serum.

A burn client is receiving treatments of topical mafenide acetate to the site of injury. The nurse monitors the client, knowing which finding indicates that a systemic effect has occurred? 1. Hyperventilation 2. Elevated blood pressure 3. Local rash at the burn site 4. Local pain at the burn site

1 Mafenide acetate is a carbonic anhydrase inhibitor and can suppress renal excretion of acid, thereby causing acidosis. Clients receiving this treatment should be monitored for signs of an acid-base imbalance (hyperventilation). If this occurs, the medication will probably be discontinued for 1 to 2 days. Options 3 and 4 describe local rather than systemic effects. Elevated blood pressure may be expected from the pain that occurs with a burn injury.

The nurse is monitoring a client admitted to the hospital with a diagnosis of appendicitis who is scheduled for surgery in 2 hours. The client begins to complain of increased abdominal pain and begins to vomit. On assessment, the nurse notes that the abdomen is distended and bowel sounds are diminished. Which is the most appropriate nursing intervention? 1. Notify the health care provider (HCP). 2. Administer the prescribed pain medication. 3. Call and ask the operating room team to perform surgery as soon as possible. 4. Reposition the client and apply a heating pad on the warm setting to the client's abdomen.

1 On the basis of the signs and symptoms presented in the question, the nurse should suspect peritonitis and notify the HCP. Administering pain medication is not an appropriate intervention. Heat should never be applied to the abdomen of a client with suspected appendicitis because of the risk of rupture. Scheduling surgical time is not within the scope of nursing practice, although the HCP probably would perform the surgery earlier than the prescheduled time.

A client has been admitted with chest trauma after a motor vehicle crash and has undergone subsequent intubation. The nurse checks the client when the high-pressure alarm on the ventilator sounds, and notes that the client has absence of breath sounds in the right upper lobe of the lung. The nurse immediately assesses for other signs of which condition? 1. Right pneumothorax 2. Pulmonary embolism 3. Displaced endotracheal tube 4. Acute respiratory distress syndrome

1 Pneumothorax is characterized by restlessness, tachycardia, dyspnea, pain with respiration, asymmetrical chest expansion, and diminished or absent breath sounds on the affected side. Pneumothorax can cause increased airway pressure because of resistance to lung inflation. Acute respiratory distress syndrome and pulmonary embolism are not characterized by absent breath sounds. An endotracheal tube that is inserted too far can cause absent breath sounds, but the lack of breath sounds most likely would be on the left side because of the degree of curvature of the right and left mainstem bronchi.

A client with diabetes mellitus visits a health care clinic. The client's diabetes mellitus previously had been well controlled with glyburide daily, but recently the fasting blood glucose level has been 180 to 200 mg/dL (10.2 to 11.4 mmol/L). Which medication, if added to the client's regimen, may have contributed to the hyperglycemia? 1. Prednisone 2. Atenolol 3. Phenelzine 4. Allopurinol

1 Prednisone may decrease the effect of oral hypoglycemics, insulin, diuretics, and potassium supplements. Option 2, a beta blocker, and option 3, a monoamine oxidase inhibitor, have their own intrinsic hypoglycemic activity. Option 4 decreases urinary excretion of sulfonylurea agents, causing increased levels of the oral agents, which can lead to hypoglycemia.

Salicylic acid is prescribed for a client with a diagnosis of psoriasis. The nurse monitors the client, knowing which finding indicates the presence of systemic toxicity from this medication? 1. Tinnitus 2. Diarrhea 3. Constipation 4. Decreased respirations

1 Salicylic acid is absorbed readily through the skin, and systemic toxicity (salicylism) can result. Symptoms include tinnitus, dizziness, hyperpnea, and psychological disturbances. Constipation and diarrhea are not associated with salicylism.

The nurse is taking the history of a client with occupational lung disease (silicosis). The nurse should assess whether the client wears which item during periods of exposure to silica particles? 1. Mask 2. Gown 3. Gloves 4. Eye protection

1 Silicosis results from chronic, excessive inhalation of particles of free crystalline silica dust. The client should wear a mask to limit inhalation of this substance, which can cause restrictive lung disease after years of exposure. Options 2, 3, and 4 are not necessary.

The nurse is caring for a client who is postoperative following a pelvic exenteration and the health care provider changes the client's diet from NPO (nothing by mouth) status to clear liquids. The nurse should check which priority item before administering the diet? 1. Bowel sounds 2. Ability to ambulate 3. Incision appearance 4. Urine specific gravity

1 The client is kept NPO until peristalsis returns, usually in 4 to 6 days. When signs of bowel function return, clear fluids are given to the client. If no distention occurs, the diet is advanced as tolerated. The most important assessment is to assess bowel sounds before feeding the client. Options 2, 3, and 4 are unrelated to the data in the question.

A client who is human immunodeficiency virus (HIV)-positive has had a tuberculin skin test (TST). The nurse notes a 7-mm area of induration at the site of the skin test and interprets the result as which finding? 1. Positive 2. Negative 3. Inconclusive 4. Need for repeat testing

1 The client with HIV infection is considered to have positive results on tuberculin skin testing with an area of induration larger than 5 mm. The client without HIV is positive with an induration larger than 10 mm. The client with HIV is immunosuppressed, making a smaller area of induration positive for this type of client. It is possible for the client infected with HIV to have false-negative readings because of the immunosuppression factor. Options 2, 3, and 4 are incorrect interpretations.

The home health care nurse is caring for a client with cancer who is complaining of acute pain. The most appropriate determination of the client's pain should include which assessment? 1. The client's pain rating 2. Nonverbal cues from the client 3. The nurse's impression of the client's pain 4. Pain relief after appropriate nursing intervention

1 The client's self-report is a critical component of pain assessment. The nurse should ask the client to describe the pain and listen carefully to the words the client uses to describe the pain. Nonverbal cues from the client are important but are not the most appropriate pain assessment measure. The nurse's impression of the client's pain is not appropriate in determining the client's level of pain. Assessing pain relief is an important measure, but this option is not related to the subject of the question.

A client has begun therapy with theophylline. The nurse should plan to teach the client to limit the intake of which items while taking this medication? 1. Coffee, cola, and chocolate 2. Oysters, lobster, and shrimp 3. Melons, oranges, and pineapple 4. Cottage cheese, cream cheese, and dairy creamers

1 Theophylline is a methylxanthine bronchodilator. The nurse teaches the client to limit the intake of xanthinecontaining foods while taking this medication. These foods include coffee, cola, and chocolate.

The home health nurse visits a client with a diagnosis of type 1 diabetes mellitus. The client relates a history of vomiting and diarrhea and tells the nurse that no food has been consumed for the last 24 hours. Which additional statement by the client indicates a need for further teaching? 1. "I need to stop my insulin." 2. "I need to increase my fluid intake." 3. "I need to monitor my blood glucose every 3 to 4 hours." 4. "I need to call the health care provider (HCP) because of these symptoms."

1 When a client with diabetes mellitus is unable to eat normally because of illness, the client still should take the prescribed insulin or oral medication. The client should consume additional fluids and should notify the HCP. The client should monitor the blood glucose level every 3 to 4 hours. The client should also monitor the urine for ketones during illness.

The nurse is teaching a client how to mix regular insulin and NPH insulin in the same syringe. Which action, if performed by the client, indicates the need for further teaching? 1. Withdraws the NPH insulin first 2. Withdraws the regular insulin first 3. Injects air into NPH insulin vial first 4. Injects an amount of air equal to the desired dose of insulin into each vial

1 When preparing a mixture of short-acting insulin, such as regular insulin, with another insulin preparation, the short-acting insulin is drawn into the syringe first. This sequence will avoid contaminating the vial of short-acting insulin with insulin of another type. Options 2, 3, and 4 identify correct actions for preparing NPH and short-acting insulin.

The nurse is reviewing the laboratory results of a client diagnosed with multiple myeloma. Which would the nurse expect to note specifically in this disorder? 1. Increased calcium level 2. Increased white blood cells 3. Decreased blood urea nitrogen level 4. Decreased number of plasma cells in the bone marrow

1 Findings indicative of multiple myeloma are an increased number of plasma cells in the bone marrow, anemia, hypercalcemia caused by the release of calcium from the deteriorating bone tissue, and an elevated blood urea nitrogen level. An increased white blood cell count may or may not be present and is not related specifically to multiple myeloma.

The home health care nurse is visiting a client who was recently diagnosed with type 2 diabetes mellitus. The client is prescribed repaglinide and metformin. The nurse should provide which instructions to the client? (SELECT ALL THAT APPLY.) 1. Diarrhea may occur secondary to the metformin. 2. The repaglinide is not taken if a meal is skipped. 3. The repaglinide is taken 30 minutes before eating. 4. A simple sugar food item is carried and used to treat mild hypoglycemia episodes. 5. Muscle pain is an expected effect of metformin and may be treated with acetaminophen. 6. Metformin increases hepatic glucose production to prevent hypoglycemia associated with repaglinide.

1, 2, 3, 4 Repaglinide, a rapid-acting oral hypoglycemic agent that stimulates pancreatic insulin secretion, should be taken before meals (approximately 30 minutes before meals) and should be withheld if the client does not eat. Hypoglycemia is a side effect of repaglinide and the client should always be prepared by carrying a simple sugar at all times. Metformin is an oral hypoglycemic given in combination with repaglinide and works by decreasing hepatic glucose production. A common side effect of metformin is diarrhea. Muscle pain may occur as an adverse effect from metformin but it might signify a more serious condition that warrants health care provider notification, not the use of acetaminophen.

The nurse is planning to teach a client with gastroesophageal reflux disease (GERD) about substances to avoid. Which items should the nurse include on this list? (SELECT ALL THAT APPLY.) 1. Coffee 2. Chocolate 3. Peppermint 4. Nonfat milk 5. Fried chicken 6. Scrambled eggs

1, 2, 3, 5 Foods that decrease lower esophageal sphincter (LES) pressure and irritate the esophagus will increase reflux and exacerbate the symptoms of GERD and therefore should be avoided. Aggravating substances include coffee, chocolate, peppermint, fried or fatty foods, carbonated beverages, and alcohol. Options 4 and 6 do not promote this effect.

The nurse is monitoring the intravenous (IV) infusion of an antineoplastic medication. During the infusion, the client complains of pain at the insertion site. On inspection of the site, the nurse notes redness and swelling and that the infusion of the medication has slowed in rate. The nurse suspects extravasation and should take which actions? (SELECT ALL THAT APPLY) 1. Stop the infusion. 2. Notify the health care provider (HCP). 3. Prepare to apply ice or heat to the site. 4. Restart the IV at a distal part of the same vein. 5. Prepare to administer a prescribed antidote into the site. 6. Increase the flow rate of the solution to flush the skin and subcutaneous tissue.

1, 2, 3, 5 Redness and swelling and a slowed infusion indicate signs of extravasation. If the nurse suspects extravasation during the IV administration of an antineoplastic medication, the infusion is stopped and the HCP is notified. Ice or heat may be prescribed for application to the site and an antidote may be prescribed to be administered into the site. Increasing the flow rate can increase damage to the tissues. Restarting an IV in the same vein can increase damage to the site and vein.

Rifabutin is prescribed for a client with active Mycobacterium avium complex (MAC) disease and tuberculosis. For which side and adverse effects of the medication should the nurse monitor? (SELECT ALL THAT APPLY.) 1. Signs of hepatitis 2. Flulike syndrome 3. Low neutrophil count 4. Vitamin B6 deficiency 5. Ocular pain or blurred vision 6. Tingling and numbness of the fingers

1, 2, 3, 5 Rifabutin may be prescribed for a client with active MAC disease and tuberculosis. It inhibits mycobacterial DNA-dependent RNA polymerase and suppresses protein synthesis. Side and adverse effects include rash, gastrointestinal disturbances, neutropenia (low neutrophil count), redorange-colored body secretions, uveitis (blurred vision and eye pain), myositis, arthralgia, hepatitis, chest pain with dyspnea, and flulike syndrome. Vitamin B6 deficiency and numbness and tingling in the extremities are associated with the use of isoniazid.

The nurse determines the client needs further instruction on cimetidine if which statements were made? (SELECT ALL THAT APPLY.) 1. "I will take the cimetidine with my meals." 2. "I'll know the medication is working if my diarrhea stops." 3. "My episodes of heartburn will decrease if the medication is effective." 4. "Taking the cimetidine with an antacid will increase its effectiveness." 5. "I will notify my health care provider if I become depressed or anxious." 6. "Some of my blood levels will need to be monitored closely since I also take warfarin for atrial fibrillation."

1, 2, 4 lleviate the symptom of heartburn, not diarrhea. Because cimetidine crosses the blood-brain barrier, central nervous system side and adverse effects, such as mental confusion, agitation, depression, and anxiety, can occur. Food reduces the rate of absorption, so if cimetidine is taken with meals, absorption will be slowed. Antacids decrease the absorption of cimetidine and should be taken at least 1 hour apart. If cimetidine is concomitantly administered with warfarin therapy, warfarin doses may need to be reduced, so prothrombin and international normalized ratio results must be followed.

A client has been diagnosed with hyperthyroidism. The nurse monitors for which signs and symptoms indicating a complication of this disorder? (SELECT ALL THAT APPLY.) 1. Fever 2. Nausea 3. Lethargy 4. Tremors 5. Confusion 6. Bradycardia

1, 2, 4, 5 : Thyroid storm is an acute and life-threatening complication that occurs in a client with uncontrollable hyperthyroidism. Signs and symptoms of thyroid storm include elevated temperature (fever), nausea, and tremors. In addition, as the condition progresses, the client becomes confused. The client is restless and anxious and experiences tachycardia. Test-Taking Strategy: Focus on the subject, signs and symptoms indicating a complication of hyperthyroidism. Recall that thyroid storm is a complication of hyperthyroidism. Options 3 and 6 can be eliminated if you remember that thyroid storm is caused by the release of thyroid hormones into the bloodstream, causing uncontrollable hyperthyroidism. Lethargy and bradycardia (think: slow down) are signs of hypothyroidism (slow metabolism).

The nurse is monitoring a client receiving levothyroxine sodium for hypothyroidism. Which findings indicate the presence of a side effect associated with this medication? (SELECT ALL THAT APPLY.) 1. Insomnia 2. Weight loss 3. Bradycardia 4. Constipation 5. Mild heat intolerance

1, 2, 5 Insomnia, weight loss, and mild heat intolerance are side effects of levothyroxine sodium. Bradycardia and constipation are not side effects associated with this medication, and rather are associated with hypothyroidism, which is the disorder that this medication is prescribed to treat.

The nurse is conducting a history and monitoring laboratory values on a client with multiple myeloma. What assessment findings should the nurse expect to note? (SELECT ALL THAT APPLY) 1. Pathological fracture 2. Urinalysis positive for nitrites 3. Hemoglobin level of 15.5 g/dL (155 mmol/L) 4. Calcium level of 8.6 mg/dL (2.15 mmol/L) 5. Serum creatinine level of 2.0 mg/dL (176.6 mcmol/L)

1, 2, 5 Multiple myeloma is a B-cell neoplastic condition characterized by abnormal malignant proliferation of plasma cells and the accumulation of mature plasma cells in the bone marrow. The client with malignant melanoma may experience pathologic fractures, hypercalcemia, anemia, recurrent infections, and renal failure. A serum calcium level of 8.6 mg/dL (2.15 mmol/L) and a hemoglobin level of 15.5 g/dL (155 mmol/L) are normal values. Therefore, the correct answers are pathological fractures, positive urinalysis for nitrites, and a serum creatinine level of 2.0 mg/dL (176.6 mcmol/L).

The nurse is reviewing the prescription for a client admitted to the hospital with a diagnosis of acute pancreatitis. Which interventions would the nurse expect to be prescribed for the client? (SELECT ALL THAT APPLY.) 1. Maintain NPO (nothing by mouth) status. 2. Encourage coughing and deep breathing. 3. Give small, frequent high-calorie feedings. 4. Maintain the client in a supine and flat position. 5. Give hydromorphone intravenously as prescribed for pain. 6. Maintain intravenous fluids at 10 mL/hour to keep the vein open.

1, 2, 5 The client with acute pancreatitis normally is placed on NPO status to rest the pancreas and suppress gastrointestinal secretions, so adequate intravenous hydration is necessary. Because abdominal pain is a prominent symptom of pancreatitis, pain medications such as morphine or hydromorphone are prescribed. Meperidine is avoided, as it may cause seizures. Some clients experience lessened pain by assuming positions that flex the trunk, with the knees drawn up to the chest. A side-lying position with the head elevated 45 degrees decreases tension on the abdomen and may help to ease the pain. The client is susceptible to respiratory infections because the retroperitoneal fluid raises the diaphragm, which causes the client to take shallow, guarded abdominal breaths. Therefore, measures such as turning, coughing, and deep breathing are instituted.

A client with carcinoma of the lung develops syndrome of inappropriate antidiuretic hormone (SIADH) as a complication of cancer. The nurse anticipates that the health care provider will request which prescriptions? (SELECT ALL THAT APPLY) 1. Radiation 2. Chemotherapy 3. Increased fluid intake 4. Decreased oral sodium intake 5. Serum sodium level determination 6. Medication that is antagonistic to antidiuretic hormone

1, 2, 5, 6 Cancer is a common cause of SIADH. In SIADH, excessive amounts of water are reabsorbed by the kidney and put into the systemic circulation. The increased water causes hyponatremia (decreased serum sodium levels) and some degree of fluid retention. The syndrome is managed by treating the condition and cause and usually includes fluid restriction, increased sodium intake, and medication with a mechanism of action that is antagonistic to antidiuretic hormone. Sodium levels are monitored closely because hypernatremia can develop suddenly as a result of treatment. The immediate institution of appropriate cancer therapy, usually radiation or chemotherapy, can cause tumor regression so that antidiuretic hormone synthesis and release processes return to normal.

A client is taking Humulin NPH insulin and regular insulin every morning. The nurse should provide which instructions to the client? (SELECT ALL THAT APPLY.) 1. Hypoglycemia may be experienced before dinnertime. 2. The insulin dose should be decreased if illness occurs. 3. The insulin should be administered at room temperature. 4. The insulin vial needs to be shaken vigorously to break up the precipitates. 5. The NPH insulin should be drawn into the syringe first, then the regular insulin.

1, 3 Humulin NPH is an intermediate-acting insulin. The onset of action is 60 to 120 minutes, it peaks in 6 to 14 hours, and its duration of action is 16 to 24 hours. Regular insulin is a short-acting insulin. Depending on the type, the onset of action is 30 to 60 minutes, it peaks in 1 to 5 hours, and its duration is 6 to 10 hours. Hypoglycemic reactions most likely occur during peak time. Insulin should be at room temperature when administered. Clients may need their insulin dosages increased during times of illness. Insulin vials should never be shaken vigorously. Regular insulin is always drawn up before NPH.

A client with a diagnosis of Addisonian crisis is being admitted to the intensive care unit. Which findings will the interprofessional health care team focus on? (SELECT ALL THAT APPLY.) 1. Hypotension 2. Leukocytosis 3. Hyperkalemia 4. Hypercalcemia 5. Hypernatremia

1, 3 In Addison's disease, also known as adrenal insufficiency, destruction of the adrenal gland leads to decreased production of adrenocortical hormones, including the glucocorticoid cortisol and the mineralocorticoid aldosterone. Addisonian crisis, also known as acute adrenal insufficiency, occurs when there is extreme physical or emotional stress and lack of sufficient adrenocortical hormones to manage the stressor. Addisonian crisis is a life-threatening emergency. One of the roles of endogenous cortisol is to enhance vascular tone and vascular response to the catecholamines epinephrine and norepinephrine. Hypotension occurs when vascular tone is decreased and blood vessels cannot respond to epinephrine and norepinephrine. The role of aldosterone in the body is to support the blood pressure by holding salt and water and excreting potassium. When there is insufficient aldosterone, salt and water are lost and potassium builds up; this leads to hypotension from decreased vascular volume, hyponatremia, and hyperkalemia. The remaining options are not associated with Addisonian crisis.

The nurse is completing an assessment on a client who is being admitted for a diagnostic workup for primary hyperparathyroidism. Which client complaint would be characteristic of this disorder? (SELECT ALL THAT APPLY.) 1. Polyuria 2. Headache 3. Bone pain 4. Nervousness 5. Weight gain

1, 3 The role of parathyroid hormone (PTH) in the body is to maintain serum calcium homeostasis. In hyperparathyroidism, PTH levels are high, which causes bone resorption (calcium is pulled from the bones). Hypercalcemia occurs with hyperparathyroidism. Elevated serum calcium levels produce osmotic diuresis and thus polyuria. This diuresis leads to dehydration (weight loss rather than weight gain). Loss of calcium from the bones causes bone pain. Options 2, 4, and 5 are not associated with hyperparathyroidism. Some gastrointestinal symptoms include anorexia, nausea, vomiting, and constipation.

A client with hyperthyroidism has been given methimazole. Which nursing considerations are associated with this medication? (SELECT ALL THAT APPLY.) 1. Administer methimazole with food. 2. Place the client on a low-calorie, low protein diet. 3. Assess the client for unexplained bruising or bleeding. 4. Instruct the client to report side and adverse effects such as sore throat, fever, or headaches. 5. Use special radioactive precautions when handling the client's urine for the first 24 hours following initial administration.

1, 3, 4 Common side effects of methimazole include nausea, vomiting, and diarrhea. To address these side effects, this medication should be taken with food. Because of the increase in metabolism that occurs in hyperthyroidism, the client should consume a high-calorie diet. Antithyroid medications can cause agranulocytosis with leukopenia and thrombocytopenia. Sore throat, fever, headache, or bleeding may indicate agranulocytosis and the health care provider should be notified immediately. Methim azole is not radioactive and should not be stopped abruptly, due to the risk of thyroid storm .

The nurse is monitoring a client who was diagnosed with type 1 diabetes mellitus and is being treated with NPH and regular insulin. Which manifestations would alert the nurse to the presence of a possible hypoglycemic reaction? (SELECT ALL THAT APPLY.) 1. Tremors 2. Anorexia 3. Irritability 4. Nervousness 5. Hot, dry skin 6. Muscle cramps

1, 3, 4 Decreased blood glucose levels produce autonomic nervous system symptoms, which are manifested classically as nervousness, irritability, and tremors. Option 5 is more likely to occur with hyperglycemia. Options 2 and 6 are unrelated to the manifestations of hypoglycemia. In hypoglycemia, usually the client feels hunger.

A client has just had a hemorrhoidectomy. Which nursing interventions are appropriate for this client? (SELECT ALL THAT APPLY.) 1. Administer stool softeners as prescribed. 2. Instruct the client to limit fluid intake to avoid urinary retention. 3. Encourage a high-fiber diet to promote bowel movements without straining. 4. Apply cold packs to the anal-rectal area over the dressing until the packing is removed. 5. Help the client to a Fowler's position to place pressure on the rectal area and decrease bleeding.

1, 3, 4 Nursing interventions after a hemorrhoidectomy are aimed at management of pain and avoidance of bleeding and incision rupture. Stool softeners and a high-fiber diet will help the client to avoid straining, thereby reducing the chances of rupturing the incision. An ice pack will increase comfort and decrease bleeding. Options 2 and 5 are incorrect interventions.

The nurse is preparing a list of home care instructions for a client who has been hospitalized and treated for tuberculosis. Which instructions should the nurse include on the list? (SELECT ALL THAT APPLY.) 1. Activities should be resumed gradually. 2. Avoid contact with other individuals, except family members, for at least 6 months. 3. Asputum culture is needed every 2 to 4 weeks once medication therapy is initiated. 4. Respiratory isolation is not necessary because family members already have been exposed. 5. Cover the mouth and nose when coughing or sneezing and put used tissues in plastic bags. 6. When 1 sputum culture is negative, the client is no longer considered infectious and usually can return to former employment.

1, 3, 4, 5 The nurse should provide the client and family with information about tuberculosis and allay concerns about the contagious aspect of the infection. The client needs to follow the medication regimen exactly as prescribed and always have a supply of the medication on hand. Side and adverse effects of the medication and ways of minimizing them to ensure compliance should be explained. After 2 to 3 weeks of medication therapy, it is unlikely that the client will infect anyone. Activities should be resumed gradually and a well-balanced diet that is rich in iron, protein, and vitamin C to promote healing and prevent recurrence of infection should be consumed. Respiratory isolation is not necessary because family members already have been exposed. Instruct the client about thorough hand washing, to cover the mouth and nose when coughing or sneezing, and to put used tissues into plastic bags. A sputum culture is needed every 2 to 4 weeks once medication therapy is initiated. When the results of 3 sputum cultures are negative, the client is no longer considered infectious and can usually return to former employment.

The community health nurse is conducting an educational session with community members regarding the signs and symptoms associated with tuberculosis. The nurse informs the participants that tuberculosis is considered as a diagnosis if which signs and symptoms are present? (SELECT ALL THAT APPLY.) 1. Dyspnea 2. Headache 3. Night sweats 4. A bloody, productive cough 5. A cough with the expectoration of mucoid sputum

1, 3, 4, 5 Tuberculosis should be considered for any clients with a persistent cough, weight loss, anorexia, night sweats, hemoptysis, shortness of breath, fever, or chills. The client's previous exposure to tuberculosis should also be assessed and correlated with the clinical manifestations.

The nurse is providing dietary teaching for a client with a diagnosis of chronic gastritis. The nurse instructs the client to include which foods rich in vitamin B12 in the diet? (SELECT ALL THAT APPLY.) 1. Nuts 2. Corn 3. Liver 4. Apples 5. Lentils 6. Bananas

1, 3, 5 Chronic gastritis causes deterioration and atrophy of the lining of the stomach, leading to the loss of function of the parietal cells. The source of intrinsic factor is lost, which results in an inability to absorb vitamin B12, leading to development of pernicious anemia. Clients must increase their intake of vitamin B12 by increasing consumption of foods rich in this vitamin, such as nuts, organ meats, dried beans, citrus fruits, green leafy vegetables, and yeast.

The nurse is admitting a client who is diagnosed with syndrome of inappropriate antidiuretic hormone secretion (SIADH) and has a serum sodium of 118 mEq/L (118 mmol/L). Which health care provider prescriptions should the nurse anticipate receiving? (SELECT ALL THAT APPLY.) 1. Initiate an infusion of 3% NaCl. 2. Administer intravenous furosemide 3. Restrict fluids to 800 mL over 24 hours. 4. Elevate the head of the bed to high Fowler's. 5. Administer a vasopressin antagonist as prescribed.

1, 3, 5 Clients with SIADH experience excess secretion of antidiuretic hormone (ADH), which leads to excess intravascular volume, a declining serum osmolarity, and dilutional hyponatremia. Management is directed at correcting the hyponatremia and preventing cerebral edema. Hypertonic saline is prescribed when the hyponatremia is severe, less than 120 mEq/L (120 mmol/L). An intravenous (IV) infusion of 3% saline is hypertonic. Hypertonic saline must be infused slowly as prescribed and an infusion pump must be used. Fluid restriction is a useful strategy aimed at correcting dilutional hyponatremia. Vasopressin is an ADH; vasopressin antagonists are used to treat SIADH. Furosemide may be used to treat extravascular volume and dilutional hyponatremia in SIADH, but it is only safe to use if the serum sodium is at least 125 mEq/L (125 mmol/L). When furosemide is used, potassium supplementation should also occur and serum potassium levels should be monitored. To promote venous return, the head of the bed should not be raised more than 10 degrees for the client with SIADH. Maximizing venous return helps to avoid stimulating stretch receptors in the heart that signal to the pituitary that more ADH is needed.

The nurse is assessing a client who is experiencing an acute episode of cholecystitis. Which of these clinical manifestations support this diagnosis? (SELECT ALL THAT APPLY.) 1. Fever 2. Positive Cullen's sign 3. Complaints of indigestion 4. Palpable mass in the left upper quadrant 5. Pain in the upper right quadrant after a fatty meal 6. Vague lower right quadrant abdominal discomfort

1, 3, 5 During an acute episode of cholecystitis, the client may complain of severe right upper quadrant pain that radiates to the right scapula or shoulder or experience epigastric pain after a fatty or high-volume meal. Fever and signs of dehydration would also be expected, as well as complaints of indigestion, belching, flatulence, nausea, and vomiting. Options 4 and 6 are incorrect because they are inconsistent with the anatomical location of the gallbladder. Option 2 (Cullen's sign) is associated with pancreatitis.

The nurse is caring for a client with lung cancer and bone metastasis. What signs and symptoms would the nurse recognize as indications of a possible oncological emergency? (SELECT ALL THAT APPLY) 1. Facial edema in the morning 2. Weight loss of 20 lb (9 kg) in 1 month 3. Serum calcium level of12 mg/dL(3.0 mmol/L) 4. Serum sodium level of 136 mg/dL (136 mmol/L) 5. Serum potassium level of 3.4 mg/dL (3.4 mmol/L) 6. Numbness and tingling of the lower extremities

1, 3, 6 Oncological emergencies include sepsis, disseminated intravascular coagulation, syndrome of inappropriate antidiuretic hormone, spinal cord compression, hypercalcemia, superior vena cava syndrome, and tumor lysis syndrome. Blockage of blood flow to the venous system of the head resulting in facial edema is a sign of superior vena cava syndrome. A serum calcium level of 12 mg/dL (3.0 mmol/L) indicates hypercalcemia. Numbness and tingling of the lower extremities could be a sign of spinal cord compression. Mild hypokalemia and weight loss are not oncological emergencies. A sodium level of 136 mg/dL (136 mmol/L) is a normal level.

The health care provider has determined that a client has contracted hepatitis A based on flulike symptoms and jaundice. Which statement made by the client supports this medical diagnosis? 1. "I have had unprotected sex with multiple partners." 2. "I ate shellfish about 2 weeks ago at a local restaurant." 3. "I was an intravenous drug abuser in the past and shared needles." 4. "I had a blood transfusion 30 years ago after major abdominal surgery."

2 : Hepatitis Ais transmitted by the fecal-oral route via contaminated water or food (improperly cooked shellfish), or infected food handlers. Hepatitis B, C, and D are transmitted most commonly via infected blood or body fluids, such as in the cases of intravenous drug abuse, history of blood transfusion, or unprotected sex with multiple partners.

A client who has been receiving radiation therapy for bladder cancer tells the nurse that it feels as if she is voiding through the vagina. The nurse interprets that the client may be experiencing which condition? 1. Rupture of the bladder 2. The development of a vesicovaginal fistula 3. Extreme stress caused by the diagnosis of cancer 4. Altered perineal sensation as a side effect of radiation therapy

2 A vesicovaginal fistula is a genital fistula that occurs between the bladder and vagina. The fistula is an abnormal opening between these 2 body parts and, if this occurs, the client may experience drainage of urine through the vagina. The client's complaint is not associated with options 1, 3, or 4.

The nurse is caring for a client after hypophysectomy and notes clear nasal drainage from the client's nostril. The nurse should take which initial action? 1. Lower the head of the bed. 2. Test the drainage for glucose. 3. Obtain a culture of the drainage. 4. Continue to observe the drainage

2 After hypophysectomy, the client should be monitored for rhinorrhea, which could indicate a cerebrospinal fluid leak. If this occurs, the drainage should be collected and tested for the presence of cerebrospinal fluid. Cerebrospinal fluid contains glucose, and if positive, this would indicate that the drainage is cerebrospinal fluid. The head of the bed should remain elevated to prevent increased intracranial pressure. Clear nasal drainage would not indicate the need for a culture. Continuing to observe the drainage without taking action could result in a serious complication.

Aclient is diagnosed with viral hepatitis, complaining of "no appetite" and "losing my taste for food." What instruction should the nurse give the client to provide adequate nutrition? 1. Select foods high in fat. 2. Increase intake of fluids, including juices. 3. Eat a good supper when anorexia is not as severe. 4. Eat less often, preferably only 3 large meals daily

2 Although no special diet is required to treat viral hepatitis, it is generally recommended that clients consume a low-fat diet, as fat may be tolerated poorly because of decreased bile production. Small, frequent meals are preferable and may even prevent nausea. Frequently, appetite is better in the morning, so it is easier to eat a good breakfast. An adequate fluid intake of 2500 to 3000 mL/day that includes nutritional juices is also important.

The nurse is preparing a plan of care for a client with diabetes mellitus who has hyperglycemia. The nurse places a priority on which client problem? 1. Lack of knowledge 2. Inadequate fluid volume 3. Compromised family coping 4. Inadequate consumption of nutrients

2 An increased blood glucose level will cause the kidneys to excrete the glucose in the urine. This glucose is accompanied by fluids and electrolytes, causing an osmotic diuresis leading to dehydration. This fluid loss must be replaced when it becomes severe. Options 1, 3, and 4 are not related specifically to the information in the question

A client with diabetes mellitus demonstrates acute anxiety when admitted to the hospital for the treatment of hyperglycemia. What is the appropriate intervention to decrease the client's anxiety? 1. Administer a sedative. 2. Convey empathy, trust, and respect toward the client. 3. Ignore the signs and symptoms of anxiety, anticipating that they will soon disappear. 4. Make sure that the client is familiar with the correct medical terms to promote understanding of what is happening.

2 Anxiety is a subjective feeling of apprehension, uneasiness, or dread. The appropriate intervention is to address the client's feelings related to the anxiety. Administering a sedative is not the most appropriate intervention and does not address the source of the client's anxiety. The nurse should not ignore the client's anxious feelings. Anxiety needs to be managed before meaningful client education can occur.

A client with acute myelocytic leukemia is being treated with busulfan. Which laboratory value would the nurse specifically monitor during treatment with this medication? 1. Clotting time 2. Uric acid level 3. Potassium level 4. Blood glucose level

2 Busulfan can cause an increase in the uric acid level. Hyperuricemia can produce uric acid nephropathy, renal stones, and acute kidney injury. Options 1, 3, and 4 are not specifically related to this medication.

The nurse is caring for a client who suffered an inhalation injury from a wood stove. The carbon monoxide blood report reveals a level of 12%. Based on this level, the nurse would anticipate noting which sign in the client? 1. Coma 2. Flushing 3. Dizziness 4. Tachycardia

2 Carbon monoxide levels between 11% and 20% result in flushing, headache, decreased visual activity, decreased cerebral functioning, and slight breathlessness; levels of 21% to 40% result in nausea, vomiting, dizziness, tinnitus, vertigo, confusion, drowsiness, pale to reddish-purple skin, and tachycardia; levels of 41% to 60% result in seizure and coma; and levels higher than 60% result in death.

A client is being admitted to the hospital for treatment of acute cellulitis of the lower left leg. During the admission assessment, the nurse expects to note which finding? 1. An inflammation of the epidermis only 2. A skin infection of the dermis and underlying hypodermis 3. An acute superficial infection of the dermis and lymphatics 4. An epidermal and lymphatic infection caused by Staphylococcus

2 Cellulitis is an infection of the dermis and underlying hypodermis that results in deep red erythema without sharp borders and spreads widely throughout tissue spaces. The skin is erythematous, edematous, tender, and sometimes nodular. Erysipelas is an acute, superficial, rapidly spreading inflammation of the dermis and lymphatics. The infection is not superficial and extends deeper than the epidermis.

The nurse is discussing the techniques of chest physiotherapy and postural drainage (respiratory treatments) to a client having expectoration problems because of chronic thick, tenacious mucus production in the lower airway. The nurse explains that after the client is positioned for postural drainage the nurse will perform which action to help loosen secretions? 1. Palpation and clubbing 2. Percussion and vibration 3. Hyperoxygenation and suctioning 4. Administer a bronchodilator and monitor peak flow

2 Chest physiotherapy of percussion and vibration helps to loosen secretions in the smaller lower airways. Postural drainage positions the client so that gravity can help mucus move from smaller airways to larger ones to support expectoration of the mucus. Options 1, 3, and 4 are not actions that will loosen secretions.

The nurse should tell the client, who is taking levothyroxine, to notify the health care provider (HCP) if which problem occurs? 1. Fatigue 2. Tremors 3. Cold intolerance 4. Excessively dry skin

2 Excessive doses of levothyroxine can produce signs and symptoms of hyperthyroidism. These include tachycardia, chest pain, tremors, nervousness, insomnia, hyperthermia, extreme heat intolerance, and sweating. The client should be instructed to notify the HCP if these occur. Options 1, 3, and 4 are signs of hypothyroidism.

A client with type 1 diabetes mellitus calls the nurse to report recurrent episodes of hypoglycemia with exercising. Which statement by the client indicates an adequate understanding of the peak action of NPH insulin and exercise? 1. "I should not exercise since I am taking insulin." 2. "The best time for me to exercise is after breakfast." 3. "The best time for me to exercise is mid- to late afternoon." 4. "NPH is a basal insulin, so I should exercise in the evening."

2 Exercise is an important part of diabetes management. It promotes weight loss, decreases insulin resistance, and helps to control blood glucose levels. Ahypoglycemic reaction may occur in response to increased exercise, so clients should exercise either an hour after mealtime or after consuming a 10- to 15-gram carbohydrate snack, and they should check their blood glucose level before exercising. Option 1 is incorrect because clients with diabetes should exercise, though they should check with their health care provider before starting a new exercise program. Option 3 in incorrect; clients should avoid exercise during the peak time of insulin. NPH insulin peaks at 4 to 12 hours; therefore, afternoon exercise takes place during the peak of the medication. Option 4 is incorrect; NPH insulin in an intermediate-acting insulin, not a basal insulin.

The nurse is caring for a client following a mastectomy. Which nursing intervention would assist in preventing lymphedema of the affected arm? 1. Placing cool compresses on the affected arm 2. Elevating the affected arm on a pillow above heart level 3. Avoiding arm exercises in the immediate postoperative period 4. Maintaining an intravenous site below the antecubital area on the affected side

2 Following mastectomy, the arm should be elevated above the level of the heart. Simple arm exercises should be encouraged. No blood pressure readings, injections, intravenous lines, or blood draws should be performed on the affected arm. Cool compresses are not a suggested measure to prevent lymphedema from occurring.

The nurse is providing medication instructions to a client with breast cancer who is receiving cyclophosphamide. The nurse should tell the client to take which action? 1. Take the medication with food. 2. Increase fluid intake to 2000 to 3000 mL daily. 3. Decrease sodium intake while taking the medication. 4. Increase potassium intake while taking the medication.

2 Hemorrhagic cystitis is an adverse effect that can occur with the use of cyclophosphamide. The client needs to be instructed to drink copious amounts of fluid during the administration of this medication. Clients also should monitor urine output for hematuria. The medication should be taken on an empty stomach, unless gastrointestinal upset occurs. Hyperkalemia can result from the use of the medication; therefore, the client would not be told to increase potassium intake. The client would not be instructed to alter sodium intake.

The nurse is preparing to suction a client via a tracheostomy tube. The nurse should plan to limit the suctioning time to a maximum of which time period? 1. 5 seconds 2. 10 seconds 3. 30 seconds 4. 60 seconds

2 Hypoxemia can be caused by prolonged suctioning, which stimulates the pacemaker cells in the heart. A vasovagal response may occur, causing bradycardia. The nurse must preoxygenate the client before suctioning and limit the suctioning pass to 10 seconds.

The nurse is assessing the perineal wound in a client who has returned from the operating room following an abdominal perineal resection and notes serosanguineous drainage from the wound. Which nursing intervention is most appropriate? 1. Clamp the surgical drain. 2. Change the dressing as prescribed. 3. Notify the health care provider (HCP). 4. Remove and replace the perineal packing

2 Immediately after surgery, profuse serosanguineous drainage from the perineal wound is expected. Therefore, the nurse should change the dressing as prescribed. A surgical drain should not be clamped because this action will cause the accumulation of drainage within the tissue. The nurse does not need to notify the HCP at this time. Drains and packing are removed gradually over a period of 5 to 7 days as prescribed. The nurse should not remove the perineal packing.

The home care nurse visits a client recently diagnosed with diabetes mellitus who is taking Humulin NPH insulin daily. The client asks the nurse how to store the unopened vials of insulin. The nurse should tell the client to take which action? 1. Freeze the insulin. 2. Refrigerate the insulin. 3. Store the insulin in a dark, dry place. 4. Keep the insulin at room temperature

2 Insulin in unopened vials should be stored under refrigeration until needed. Vials should not be frozen. When stored unopened under refrigeration, insulin can be used up to the expiration date on the vial. Options 1, 3, and 4 are incorrect.

The nurse is providing care for a client with a recent transverse colostomy. Which observation requires immediate notification of the health care provider? 1. Stoma is beefy red and shiny 2. Purple discoloration of the stoma 3. Skin excoriation around the stoma 4. Semi-formed stool noted in the ostomy pouch

2 Ischemia of the stoma would be associated with a dusky or bluish or purple color. A beefy red and shiny stoma is normal and expected. Skin excoriation needs to be addressed and treated but does not require as immediate attention as purple discoloration of the stoma. Semi-formed stool is a normal finding.

A client has been taking isoniazid for 2 months. The client complains to the nurse about numbness, paresthesias, and tingling in the extremities. The nurse interprets that the client is experiencing which problem? 1. Hypercalcemia 2. Peripheral neuritis 3. Small blood vessel spasm 4. Impaired peripheral circulation

2 Isoniazid is an antitubercular medication. A common side effect of isoniazid is peripheral neuritis, manifested by numbness, tingling, and paresthesias in the extremities. This can be minimized with pyridoxine (vitamin B6) intake. Options 1, 3, and 4 are not associated with the information in the question.

A client is to begin a 6-month course of therapy with isoniazid. The nurse should plan to teach the client to take which action? 1. Use alcohol in small amounts only. 2. Report yellow eyes or skin immediately. 3. Increase intake of Swiss or aged cheeses. 4. Avoid vitamin supplements during therapy.

2 Isoniazid is hepatotoxic, and therefore the client is taught to report signs and symptoms of hepatitis immediately, which include yellow skin and sclera. For the same reason, alcohol should be avoided during therapy. The client should avoid intake of Swiss cheese, fish such as tuna, and foods containing tyramine because they may cause a reaction characterized by redness and itching of the skin, flushing, sweating, tachycardia, headache, or lightheadedness. The client can avoid developing peripheral neuritis by increasing the intake of pyridoxine (vitamin B6) during the course of isoniazid therapy.

Isotretinoin is prescribed for a client with severe acne. Before the administration of this medication, the nurse anticipates which laboratory test will be prescribed? 1. Potassium level 2. Triglyceride level 3. Hemoglobin A1C 4. Total cholesterol level

2 Isotretinoin can elevate triglyceride levels. Blood triglyceride levels should be measured before treatment and periodically thereafter until the effect on the triglycerides has been evaluated. There is no indication that isotretinoin affects potassium, hemoglobin A1C, or total cholesterol levels

An oxygen delivery system is prescribed for a client with chronic obstructive pulmonary disease to deliver a precise oxygen concentration. Which oxygen delivery system would the nurse prepare for the client? 1. Face tent 2. Venturi mask 3. Aerosol mask 4. Tracheostomy collar

2 The Venturi mask delivers the most accurate oxygen concentration. It is the best oxygen delivery system for the client with chronic airflow limitation such as chronic obstructive pulmonary disease, because it delivers a precise oxygen concentration. The face tent, aerosol mask, and tracheostomy collar are also high-flow oxygen delivery systems but most often are used to administer high humidity.

The nurse is providing discharge teaching for a client newly diagnosed with type 2 diabetes mellitus who has been prescribed metformin. Which client statement indicates the need for further teaching? 1. "It is okay if I skip meals now and then." 2. "I need to constantly watch for signs of low blood sugar." 3. "I need to let my health care provider know if I get unusually tired." 4. "I will be sure to not drink alcohol excessively while on this medication."

2 Metformin is classified as a biguanide and is the most commonly used medication for type 2 diabetes mellitus initially. It is also often used as a preventive medication for those at high risk for developing diabetes mellitus. When used alone, metformin lowers the blood sugar after meal intake as well as fasting blood glucose levels. Metformin does not stimulate insulin release and therefore poses little risk for hypoglycemia. For this reason, metformin is well suited for clients who skip meals. Unusual somnolence, as well as hyperventilation, myalgia, and malaise, are early signs of lactic acidosis, a toxic effect associated with metformin. If any of these signs or symptoms occur, the client should inform the health care provider immediately. While it is best to avoid consumption of alcohol, it is not always realistic or feasible for clients to quit drinking altogether; for this reason, clients should be informed that excessive alcohol intake can cause an adverse reaction with metformin

A client is admitted to an emergency department, and a diagnosis of myxedema coma is made. Which action should the nurse prepare to carry out initially? 1. Warm the client. 2. Maintain a patent airway. 3. Administer thyroid hormone. 4. Administer fluid replacement.

2 Myxedema coma is a rare but serious disorder that results from persistently low thyroid production. Coma can be precipitated by acute illness, rapid withdrawal of thyroid medication, anesthesia and surgery, hypothermia, and the use of sedatives and opioid analgesics. In myxedema coma, the initial nursing action is to maintain a patent airway. Oxygen should be administered, followed by fluid replacement, keeping the client warm, monitoring vital signs, and administering thyroid hormones by the intravenous route.

A client has been taking omeprazole for 4 weeks. The ambulatory care nurse evaluates that the client is receiving the optimal intended effect of the medication if the client reports the absence of which symptom? 1. Diarrhea 2. Heartburn 3. Flatulence 4. Constipation

2 Omeprazole is a proton pump inhibitor classified as an antiulcer agent. The intended effect of the medication is relief of pain from gastric irritation, often called heartburn by clients. Omeprazole is not used to treat the conditions identified in options 1, 3, and 4.

The nurse is providing instructions to the client newly diagnosed with diabetes mellitus who has been prescribed pramlintide. Which instruction should the nurse include in the discharge teaching? 1. "Inject the pramlintide at the same time you take your other medications." 2. "Take your prescribed pills 1 hour before or 2 hours after the injection." 3. "Be sure to take the pramlintide with food so you don't upset your stomach." 4. "Make sure you take your pramlintide immediately after you eat so you don't experience a low blood sugar."

2 Pramlintide is used for clients with types 1 and 2 diabetes mellitus who use insulin. It is administered subcutaneously before meals to lower blood glucose level after meals, leading to less fluctuation during the day and better long-term glucose control. Because pramlintide delays gastric emptying, oral medications should be given 1 hour before or 2 hours after an injection of pramlintide; therefore, instructing the client to take his or her pills 1 hour before or 2 hours after the injection is correct. Pramlintide should not be taken at the same time as other medications. Pramlintide is given immediately before the meal in order to control postprandial rise in blood glucose, not necessarily to prevent stomach upset. It is incorrect to instruct the client to take the medication after eating, as it will not achieve its full therapeutic effect.

The nurse has a prescription to give a client salmeterol, 2 puffs, and beclomethasone dipropionate, 2 puffs, by metered-dose inhaler. The nurse should administer the medication using which procedure? 1. Beclomethasone first and then the salmeterol 2. Salmeterol first and then the beclomethasone 3. Alternating a single puff of each, beginning with the salmeterol 4. Alternating a single puff of each, beginning with the beclomethasone

2 Salmeterol is an adrenergic type of bronchodilator and beclomethasone dipropionate is a glucocorticoid. Bronchodilators are always administered before glucocorticoids when both are to be given on the same time schedule. This allows for widening of the air passages by the bronchodilator, which then makes the glucocorticoid more effective.

The nurse is administering fluids intravenously as prescribed to a client who sustained superficial partial-thickness burn injuries of the back and legs. In evaluating the adequacy of fluid resuscitation, the nurse understands that which assessment would provide the most reliable indicator for determining the adequacy? 1. Vital signs 2. Urine output 3. Mental status 4. Peripheral pulses

2 Successful or adequate fluid resuscitation in the client is signaled by stable vital signs, adequate urine output, palpable peripheral pulses, and clear sensorium. However, the most reliable indicator for determining the adequacy of fluid resuscitation, especially in a client with burns, is the urine output. For an adult, the hourly urine volume should be 30 to 50 mL.

A client with metastatic breast cancer is receiving tamoxifen. The nurse specifically monitors which laboratory value while the client is taking this medication? 1. Glucose level 2. Calcium level 3. Potassium level 4. Prothrombin time

2 Tamoxifen may increase calcium, cholesterol, and triglyceride levels. Before the initiation of therapy, a complete blood count, platelet count, and serum calcium level should be assessed. These blood levels, along with cholesterol and triglyceride levels, should be monitored periodically during therapy. The nurse should assess for hypercalcemia while the client is taking this medication. Signs of hypercalcemia include increased urine volume, excessive thirst, nausea, vomiting, constipation, hypotonicity of muscles, and deep bone and flank pain.

The nurse is instructing a client to perform testicular self-examination (TSE). The nurse should provide the client with which information about the procedure? 1. To examine the testicles while lying down 2. That the best time for the examination is after a shower 3. To gently feel the testicle with 1 finger to feel for a growth 4. That TSEs should be done at least every 6 months

2 The TSE is recommended monthly after a warm bath or shower when the scrotal skin is relaxed. The client should stand to examine the testicles. Using both hands, with fingers under the scrotum and thumbs on top, the client should gently roll the testicles, feeling for any lumps

A client who uses nonsteroidal anti-inflammatory drugs (NSAIDs) has been taking misoprostol. The nurse determines that the misoprostol is having the intended therapeutic effect if which finding is noted? 1. Resolved diarrhea 2. Relief of epigastric pain 3. Decreased platelet count 4. Decreased white blood cell count

2 The client who uses NSAIDs is prone to gastric mucosal injury. Misoprostol is a gastric protectant and is given specifically to prevent this occurrence in clients taking NSAIDs frequently. Diarrhea can be a side effect of the medication but is not an intended effect. Options 3 and 4 are unrelated to the purpose of misoprostol.

The nurse is reviewing the history of a client with bladder cancer. The nurse expects to note documentation of which most common sign or symptom of this type of cancer? 1. Dysuria 2. Hematuria 3. Urgency on urination 4. Frequency of urination

2 The most common sign in clients with cancer of the bladder is hematuria. The client also may experience irritative voiding symptoms such as frequency, urgency, and dysuria, and these symptoms often are associated with carcinoma in situ. Dysuria, urgency, and frequency of urination are also symptoms of a bladder infection.

The nurse is preparing to give a bed bath to an immobilized client with tuberculosis. The nurse should wear which items when performing this care? 1. Surgical mask and gloves 2. Particulate respirator, gown, and gloves 3. Particulate respirator and protective eyewear 4. Surgical mask, gown, and protective eyewear

2 The nurse who is in contact with a client with tuberculosis should wear an individually fitted particulate respirator. The nurse also would wear gloves as per standard precautions. The nurse wears a gown when the possibility exists that the clothing could become contaminated, such as when giving a bed bath.

A client with Crohn's disease is scheduled to receive an infusion of infliximab. What intervention by the nurse will determine the effectiveness of treatment? 1. Monitoring the leukocyte count for 2 days after the infusion 2. Checking the frequency and consistency of bowel movements 3. Checking serum liver enzyme levels before and after the infusion 4. Carrying out a Hematest on gastric fluids after the infusion is completed

2 The principal manifestations of Crohn's disease are diarrhea and abdominal pain. Infliximab is an immunomodulator that reduces the degree of inflammation in the colon, thereby reducing the diarrhea. Options 1, 3, and 4 are unrelated to this medication.

The emergency department nurse is assessing a client who has sustained a blunt injury to the chest wall. Which finding indicates the presence of a pneumothorax in this client? 1. A low respiratory rate 2. Diminished breath sounds 3. The presence of a barrel chest 4. A sucking sound at the site of injury

2 This client has sustained a blunt or closed-chest injury. Basic symptoms of a closed pneumothorax are shortness of breath and chest pain. A larger pneumothorax may cause tachypnea, cyanosis, diminished breath sounds, and subcutaneous emphysema. Hyperresonance also may occur on the affected side. A sucking sound at the site of injury would be noted with an open chest injury.

The clinic nurse notes that the health care provider has documented a diagnosis of herpes zoster (shingles) in the client's chart. Based on an understanding of the cause of this disorder, the nurse determines that this definitive diagnosis was made by which diagnostic test? 1. Positive patch test 2. Positive culture results 3. Abnormal biopsy results 4. Wood's light examination indicative of infection

2 With the classic presentation of herpes zoster, the clinical examination is diagnostic. However, viral culture of the lesion provides a definitive diagnosis. Herpes zoster (shingles) is caused by a reactivation of the varicella-zoster virus, the virus that causes chickenpox. A patch test is a skin test that involves the administration of an allergen to the surface of the skin to identify specific allergies. A biopsy would provide a cytological examination of tissue. In a Wood's light examination, the skin is viewed under ultraviolet light to identify superficial infections of the skin.

The nurse is caring for a client hospitalized with acute exacerbation of chronic obstructive pulmonary disease. Which findings would the nurse expect to note on assessment of this client? (SELECT ALL THAT APPLY.) 1. A low arterial PCo2 level 2. Ahyperinflated chest noted on the chest x-ray 3. Decreased oxygen saturation with mild exercise 4. A widened diaphragm noted on the chest x-ray 5. Pulmonary function tests that demonstrate increased vital capacity

2, 3 Clinical manifestations of chronic obstructive pulmonary disease (COPD) include hypoxemia, hypercapnia, dyspnea on exertion and at rest, oxygen desaturation with exercise, and the use of accessory muscles of respiration. Chest x-rays reveal a hyperinflated chest and a flattened diaphragm if the disease is advanced. Pulmonary function tests will demonstrate decreased vital capacity.

A client returns to the clinic for follow-up treatment following a skin biopsy of a suspicious lesion performed 1 week ago. The biopsy report indicates that the lesion is a melanoma. The nurse understands that melanoma has which characteristics? (SELECT ALL THAT APPLY.) 1. Lesion is painful to touch. 2. Lesion is highly metastatic. 3. Lesion is a nevus that has changes in color. 4. Skin under the lesion is reddened and warm to touch. 5. Lesion occurs in body area exposed to outdoor sunlight.

2, 3 Melanomas are pigmented malignant lesions originating in the melanin-producing cells of the epidermis. Melanomas cause changes in a nevus (mole), including color and borders. This skin cancer is highly metastatic, and a person's survival depends on early diagnosis and treatment. Melanomas are not painful or accompanied by sign of inflammation. Although sun exposure increases the risk of melanoma, lesions are most commonly found on the upper back and legs and on the soles and palms of persons with dark skin.

When caring for a client with an internal radiation implant, the nurse should observe which principles? (SELECT ALL THAT APPLY.) 1. Limiting the time with the client to 1 hour per shift. 2. Keeping pregnant women out of the client's room. 3. Placing the client in a private room with a private bath. 4. Wearing a lead shield when providing direct client care. 5. Removing the dosimeter film badge when entering the client's room. 6. Allowing individuals younger than 16 years old in the room as long as they are 6 feet away from the client.

2, 3, 4 The time that the nurse spends in the room of a client with an internal radiation implant is 30 minutes per 8-hour shift. The client must be placed in a private room with a private bath. Lead shielding can be used to reduce the transmission of radiation. The dosimeter film badge must be worn when in the client's room. Children younger than 16 years of age and pregnant women are not allowed in the client's room.

A client is diagnosed as having a bowel tumor. The nurse should monitor the client for which complications of this type of tumor? (SELECT ALL THAT APPLY) 1. Flatulence 2. Peritonitis 3. Hemorrhage 4. Fistula formation 5. Bowel perforation 6. Lactose intolerance

2, 3, 4, 5 Complications of bowel tumors include bowel perforation, which can result in hemorrhage and peritonitis. Other complications include bowel obstruction and fistula formation. Flatulence can occur but is not a complication; lactose intolerance also is not a complication of intestinal tumor.

A client with a diagnosis of diabetic ketoacidosis (DKA) is being treated in the emergency department. Which findings support this diagnosis? (SELECT ALL THAT APPLY.) 1. Increase in pH 2. Comatose state 3. Deep, rapid breathing 4. Decreased urine output 5. Elevated blood glucose level

2, 3, 5 Because of the profound deficiency of insulin associated with DKA, glucose cannot be used for energy and the body breaks down fat as a secondary source of energy. Ketones, which are acid byproducts of fat metabolism, build up and the client experiences a metabolic ketoacidosis. High serum glucose contributes to an osmotic diuresis and the client becomes severely dehydrated. If untreated, the client will become comatose due to severe dehydration, acidosis, and electrolyte imbalance. Kussmaul's respirations, the deep rapid breathing associated with DKA, is a compensatory mechanism by the body. The body attempts to correct the acidotic state by blowing off carbon dioxide (CO2), which is an acid. In the absence of insulin, the client will experience severe hyperglycemia. Option 1 is incorrect because in acidosis the pH would be low. Option 4 is incorrect because a high serum glucose will result in an osmotic diuresis and the client will experience polyuria.

The nurse teaches a client with diabetes mellitus about differentiating between hypoglycemia and ketoacidosis. The client demonstrates an understanding of the teaching by stating that a form of glucose should be taken if which symptom or symptoms develop? (SELECT ALL THAT APPLY.) 1. Polyuria 2. Shakiness 3. Palpitations 4. Blurred vision 5. Lightheadedness 6. Fruity breath odor

2, 3, 5 Shakiness, palpitations, and lightheadedness are signs/symptoms of hypoglycemia and would indicate the need for food or glucose. Polyuria, blurred vision, and a fruity breath odor are manifestations of hyperglycemia.

The health education nurse provides instructions to a group of clients regarding measures that will assist in preventing skin cancer. Which instructions should the nurse provide? (SELECT ALL THAT APPLY) 1. Sunscreen should be applied every 8 hours. 2. Use sunscreen when participating in outdoor activities 3. Wear a hat, opaque clothing, and sunglasses when in the sun. 4. Avoid sun exposure in the late afternoon and early evening hours. 5. Examine your body monthly for any lesions that may be suspicious.

2, 3, 5 The client should be instructed to avoid sun exposure between the hours of brightest sunlight: 10 a.m. and 4 p.m. Sunscreen, a hat, opaque clothing, and sunglasses should be worn for outdoor activities. The client should be instructed to examine the body monthly for the appearance of any cancerous or any precancerous lesions. Sunscreen should be reapplied every 2 to 3 hours and after swimming or sweating; otherwise, the duration of protection is reduced.

Glimepiride is prescribed for a client with diabetes mellitus. The nurse instructs the client that which food items are most acceptable to consume while taking this medication? (SELECT ALL THAT APPLY.) 1. Alcohol 2. Red meats 3. Whole-grain cereals 4. Low-calorie desserts 5. Carbonated beverages

2, 3, 5 When alcohol is combined with glimepiride, a disulfiram-like reaction may occur. This syndrome includes flushing, palpitations, and nausea. Alcohol can also potentiate the hypoglycem ic effects of the m edication. Clients need to be instructed to avoid alcohol consumption while taking this medication. Low-calorie desserts should also be avoided. Even though the calorie content may be low, carbohydrate content is m ost likely high and can affect the blood glucose. The items in options 2, 3, and 5 are acceptable to consume.

A client is brought to the emergency department with partial-thickness burns to his face, neck, arms, and chest after trying to put out a car fire. The nurse should implement which nursing actions for this client? (SELECT ALL THAT APPLY.) 1. Restrict fluids. 2. Assess for airway patency. 3. Administer oxygen as prescribed. 4. Place a cooling blanket on the client. 5. Elevate extremities if no fractures are present. 6. Prepare to give oral pain medication as prescribed.

2, 3, 5 The primary goal for a burn injury is to maintain a patent airway, administer intravenous (IV) fluids to prevent hypovolemic shock, and preserve vital organ functioning. Therefore, the priority actions are to assess for airway patency and maintain a patent airway. The nurse then prepares to administer oxygen. Oxygen is necessary to perfuse vital tissues and organs. An IV line should be obtained and fluid resuscitation started. The extremities are elevated to assist in preventing shock and decrease fluid moving to the extremities, especially in the burn-injured upper extremities. The client is kept warm since the loss of skin integrity causes heat loss. The client is placed on NPO (nothing by mouth) status because of the altered gastrointestinal function that occurs as a result of a burn injury.

The nurse is monitoring a client diagnosed with acromegaly who was treated with transsphenoidal hypophysectomy and is recovering in the intensive care unit. Which findings should alert the nurse to the presence of a possible postoperative complication? (SELECT ALL THAT APPLY.) 1. Anxiety 2. Leukocytosis 3. Chvostek's sign 4. Urinary output of 800 mL/hour 5. Clear drainage on nasal dripper pad

2, 4, 5 Acromegaly results from excess secretion of growth hormone, usually caused by a benign tumor on the anterior pituitary gland. Treatment is surgical removal of the tumor, usually with a sublingual transsphenoidal complete or partial hypophysectomy. The sublingual transsphenoidal approach is often through an incision in the inner upper lip at the gum line. Transsphenoidal surgery is a type of brain surgery and infection is a primary concern. Leukocytosis, or an elevated white count, may indicate infection. Diabetes insipidus is a possible complication of transsphenoidal hypophysectomy. In diabetes insipidus there is decreased secretion of antidiuretic hormone and clients excrete large amounts of dilute urine. Following transsphenoidal surgery, the nasal passages are packed and a dripper pad is secured under the nares. Clear drainage on the dripper pad is suggestive of a cerebrospinal fluid leak. The surgeon should be notified and the drainage should be tested for glucose. A cerebrospinal fluid leak increases the postoperative risk of meningitis. Anxiety is a nonspecific finding that is common to many disorders. Chvostek's sign is a test of nerve hyperexcitability associated with hypocalcemia and is seen as grimacing in response to tapping on the facial nerve. Chvostek's sign has no association with complications of sublingual transsphenoidal hypophysectomy

The nurse teaches the client, who is newly diagnosed with diabetes insipidus, about the prescribed intranasal desmopressin. Which statements by the client indicate understanding? (SELECT ALL THAT APPLY.) 1. "This medication will turn my urine orange." 2. "I should decrease my oral fluids when I start this medication." 3. "The amount of urine I make should increase if this medicine is working." 4. "I need to follow a low-fat diet to avoid pancreatitis when taking this medicine." 5. "I should report headache and drowsiness to my health care provider since these symptoms could be related to my desmopressin."

2, 5 In diabetes insipidus, there is a deficiency in antidiuretic hormone (ADH), resulting in large urinary losses. Desmopressin is an analog of ADH. Clients with diabetes insipidus drink high volumes of fluid (polydipsia) as a compensatory mechanism to counteract urinary losses and maintain fluid balance. Once desmopressin is started, oral fluids should be decreased to prevent water intoxication. Therefore, clients with diabetes insipidus should decrease their oral fluid intake when they start desmopressin. Headache and drowsiness are signs of water intoxication in the client taking desmopressin and should be reported to the health care provider. Desmopressin does not turn urine orange. The amount of urine should decrease, not increase, when desmopressin is started. Desmopressin does not cause pancreatitis

The low-pressure alarm sounds on a ventilator. The nurse assesses the client and then attempts to determine the cause of the alarm. If unsuccessful in determining the cause of the alarm, the nurse should take what initial action? 1. Administer oxygen 2. Check the client's vital signs 3. Ventilate the client manually 4. Start cardiopulmonary resuscitation

3 If at any time an alarm is sounding and the nurse cannot quickly ascertain the problem, the client is disconnected from the ventilator and manual resuscitation is used to support respirations until the problem can be corrected. No reason is given to begin cardiopulmonary resuscitation. Checking vital signs is not the initial action. Although oxygen is helpful, it will not provide ventilation to the client.

An adult client was burned in an explosion. The burn initially affected the client's entire face (anterior half of the head) and the upper half of the anterior torso, and there were circumferential burns to the lower half of both arms. The client's clothes caught on fire, and the client ran, causing subsequent burn injuries to the posterior surface of the head and the upper half of the posterior torso. Using the rule of nines, what would be the extent of the burn injury? 1. 18% 2. 24% 3. 36% 4. 48%

3 According to the rule of nines, with the initial burn, the anterior half of the head equals 4.5%, the upper half of the anterior torso equals 9%, and the lower half of both arms equals 9%. The subsequent burn included the posterior half of the head, equaling 4.5%, and the upper half of posterior torso, equaling 9%. This totals 36%.

The nurse is reviewing the record of a client with a diagnosis of cirrhosis and notes that there is documentation of the presence of asterixis. How should the nurse assess for its presence? 1. Dorsiflex the client's foot. 2. Measure the abdominal girth. 3. Ask the client to extend the arms. 4. Instruct the client to lean forward.

3 Asterixis is irregular flapping movements of the fingers and wrists when the hands and arms are outstretched, with the palms down, wrists bent up, and fingers spread. Asterixis is the most common and reliable sign that hepatic encephalopathy is developing. Options 1, 2, and 4 are incorrect.

The nurse is caring for a client following an autograft and grafting to a burn wound on the right knee. What would the nurse anticipate to be prescribed for the client? 1. Out-of-bed activities 2. Bathroom privileges 3. Immobilization of the affected leg 4. Placing the affected leg in a dependent position

3 Autografts placed over joints or on the lower extremities after surgery often are elevated and immobilized for 3 to 7 days. This period of immobilization allows the autograft time to adhere to the wound bed. Getting out of bed, going to the bathroom, and placing the grafted leg dependent would put stress on the grafted wound.

A clinic nurse prepares a teaching plan for a client receiving an antineoplastic medication. When implementing the plan, the nurse should make which statement to the client? 1. "You can take aspirin as needed for headache." 2. "You can drink beverages containing alcohol in moderate amounts each evening." 3. "You need to consult with the health care provider (HCP) before receiving immunizations." 4. "It is fine to receive a flu vaccine at the local health fair without HCP approval because the flu is so contagious."

3 Because antineoplastic medications lower the resistance of the body, clients must be informed not to receive immunizations without the HCP's approval. Clients also need to avoid contact with individuals who have recently received a live virus vaccine. Clients need to avoid aspirin and aspirin-containing products to minimize the risk of bleeding, and theyneed to avoid alcohol to minimize the risk of toxicity and side/adverse effects

The nurse is monitoring the laboratory results of a client receiving an antineoplastic medication by the intravenous route. The nurse plans to initiate bleeding precautions if which laboratory result is noted? 1. A clotting time of 10 minutes 2. An ammonia level of 10 mcg/dL (6 mcmol/L). 3. A platelet count of 50,000 mm3 (50 Â 109 /L) 4. A white blood cell count of 5000 mm3 (5.0 Â 109 /L)

3 Bleeding precautions need to be initiated when the platelet count decreases. The normal platelet count is 150,000 to 450,000 mm3 (150-400 Â 109 /L). When the platelet count decreases, the client is at risk for bleeding. The normal white blood cell count is 5000 to 10,000 mm3 (5.0-10.0Â109/L). When the white blood cell count drops, neutropenic precautions need to be implemented. The normal clotting time is 8 to 15 minutes. The normal ammonia value is 10 to 80 mcg/dL (6-47 mcmol/L).

The nurse has given instructions to a client who has just been prescribed cholestyramine. Which statement by the client indicates a need for further instruction? 1. "I will continue taking vitamin supplements." 2. "This medication will help to lower my cholesterol." 3. "This medication should only be taken with water." 4. "A high-fiber diet is important while taking this medication."

3 Cholestyramine is a bile acid sequestrant used to lower the cholesterol level, and client compliance is a problem because of its taste and palatability. The use of flavored products or fruit juices can improve the taste. Some side effects of bile acid sequestrants include constipation and decreased vitamin absorption.

An older client recently has been taking cimetidine. The nurse monitors the client for which most frequent central nervous system side effect of this medication? 1. Tremors 2. Dizziness 3. Confusion 4. Hallucinations

3 Cimetidine is a histamine (H2)-receptor antagonist. Older clients are especially susceptible to central nervous system side effects of cimetidine. The most frequent of these is confusion. Less common central nervous system side effects include headache, dizziness, drowsiness, and hallucinations

The nurse is reviewing the laboratory results for a client with cirrhosis and notes that the ammonia level is 85 mcg/dL (51 mcmol/L). Which dietary selection does the nurse suggest to the client? 1. Roast pork 2. Cheese omelet 3. Pasta with sauce 4. Tuna fish sandwich

3 Cirrhosis is a chronic, progressive disease of the liver characterized by diffuse degeneration and destruction of hepatocytes. The serum ammonia level assesses the ability of the liver to deaminate protein byproducts. Normal reference interval is 10 to 80 mcg/dL (6 to 47 mcmol/L). Most of the ammonia in the body is found in the gastrointestinal tract. Protein provided by the diet is transported to the liver by the portal vein. The liver breaks down protein, which results in the formation of ammonia. Foods high in protein should be avoided since the client's ammonia level is elevated above the normal range; therefore, pasta with sauce would be the best selection.

Adaily dose of prednisone is prescribed for a client. The nurse provides instructions to the client regarding administration of the medication and should instruct the client that which time is best to take this medication? 1. At noon 2. At bedtime 3. Early morning 4. Any time, at the same time, each day

3 Corticosteroids (glucocorticoids) should be administered before 9 a.m. Administration at this time helps to minimize adrenal insufficiency and mimics the burst of glucocorticoids released naturally by the adrenal glands each morning. Options 1, 2, and 4 are incorrect.

The nurse is providing discharge instructions to a client following gastrectomy and should instruct the client to take which measure to assist in preventing dumping syndrome? 1. Ambulate following a meal. 2. Eat high-carbohydrate foods. 3. Limit the fluids taken with meals. 4. Sit in a high Fowler's position during meals

3 Dumping syndrome is a term that refers to a constellation of vasomotor symptoms that occurs after eating, especially following a gastrojejunostomy (Billroth II procedure). Early manifestations usually occur within 30 minutes of eating and include vertigo, tachycardia, syncope, sweating, pallor, palpitations, and the desire to lie down. The nurse should instruct the client to decrease the amount of fluid taken at meals and to avoid high-carbohydrate foods, including fluids such as fruit nectars; to assume a low Fowler's position during meals; to lie down for 30 minutes after eating to delay gastric emptying; and to take antispasmodics as prescribed.

The nurse is suctioning a client via an endotracheal tube. During the suctioning procedure, the nurse notes on the monitor that the heart rate is decreasing. Which nursing intervention is appropriate? 1. Continue to suction. 2. Notify the health care provider immediately. 3. Stop the procedure and reoxygenate the client. 4. Ensure that the suction is limited to 15 seconds

3 During suctioning, the nurse should monitor the client closely for adverse effects, including hypoxemia, cardiac irregularities such as a decrease in heart rate resulting from vagal stimulation, mucosal trauma, hypotension, and paroxysmal coughing. If adverse effects develop, especially cardiac irregularities, the procedure is stopped and the client is re-oxygenated.

As part of chemotherapy education, the nurse teaches a female client about the risk for bleeding and self-care during the period of greatest bone marrow suppression (the nadir). The nurse understands that further teaching is needed if the client makes which statement? 1. "I should avoid blowing my nose." 2. "I may need a platelet transfusion if my platelet count is too low." 3. "I'm going to take aspirin for my headache as soon as I get home." 4. "I will count the number of pads and tampons I use when menstruating."

3 During the period of greatest bone marrow suppression (the nadir), the platelet count may be low, less than 20,000 cells mm3 (20.0 Â 109 /L). The correct option describes an incorrect statement by the client. Aspirin and nonsteroidal anti-inflammatory drugs and products that contain aspirin should be avoided because of their antiplatelet activity. Options 1, 2, and 4 are correct statements by the client to prevent and monitor bleeding.

The nurse is caring for a client following a gastrojejunostomy (Billroth II procedure). Which postoperative prescription should the nurse question and verify? 1. Leg exercises 2. Early ambulation 3. Irrigating the nasogastric tube 4. Coughing and deep-breathing exercises

3 In a gastrojejunostomy (Billroth II procedure), the proximal remnant of the stomach is anastomosed to the proximal jejunum. Patency of the nasogastric tube is critical for preventing the retention of gastric secretions. The nurse should never irrigate or reposition the gastric tube after gastric surgery, unless specifically prescribed by the health care provider. In this situation, the nurse should clarify the prescription. Options 1, 2, and 4 are appropriate postoperative interventions.

A client is admitted to a hospital with a diagnosis of diabetic ketoacidosis (DKA). The initial blood glucose level is 950 mg/dL (54.2 mmol/L). A continuous intravenous (IV) infusion of short-acting insulin is initiated, along with IV rehydration with normal saline. The serum glucose level is now decreased to 240 mg/dL (13.7 mmol/L). The nurse would next prepare to administer which medication? 1. An ampule of 50% dextrose 2. NPH insulin subcutaneously 3. IV fluids containing dextrose 4. Phenytoin for the prevention of seizures.

3 Emergency management of DKA focuses on correcting fluid and electrolyte imbalances and normalizing the serum glucose level. If the corrections occur too quickly, serious consequences, including hypoglycemia and cerebral edema, can occur. During the management of DKA, when the blood glucose level falls to 250 to 300 mg/dL (14.2 to 17.1 mmol/L), the IV infusion rate is reduced and a dextrose solution is added to maintain a blood glucose level of about 250 mg/dL (14.2 mmol/L), or until the client recovers from ketosis. Fifty percent of dextrose is used to treat hypoglycemia. NPH insulin is not used to treat DKA. Phenytoin is not a usual treatment measure for DKA

A client with a chest injury has suffered flail chest. The nurse assesses the client for which most distinctive sign of flail chest? 1. Cyanosis 2. Hypotension 3. Paradoxical chest movement 4. Dyspnea, especially on exhalation

3 Flail chest results from multiple rib fractures. This results in a "floating" section of ribs. Because this section is unattached to the rest of the bony rib cage, this segment results in paradoxical chest movement. This means that the force of inspiration pulls the fractured segment inward, while the rest of the chest expands. Similarly, during exhalation, the segment balloons outward while the rest of the chest moves inward. This is a characteristic sign of flail chest

A client is undergoing fluid replacement after being burned on 20% of her body 12 hours ago. The nursing assessment reveals a blood pressure of 90/50 mmHg, a pulse rate of 110 beats/minute, and a urine output of 20 mL over the past hour. The nurse reports the findings to the health care provider (HCP) and anticipates which prescription? 1. Transfusing 1 unit of packed red blood cells 2. Administering a diuretic to increase urine output 3. Increasing the amount of intravenous (IV) lactated Ringer's solution administered per hour 4. Changing the IV lactated Ringer's solution to one that contains 5% dextrose in water

3 Fluid management during the first 24 hours following a burn injury generally includes the infusion of (usually) lactated Ringer's solution. Lactated Ringer's solution is an isotonic solution that contains electrolytes that will maintain fluid volume in the circulation. Fluid resuscitation is determined by urine output and hourly urine output should be at least 30 mL/ hour. The client's urine output is indicative of insufficient fluid resuscitation, which places the client at risk for inadequate perfusion of the brain, heart, kidneys, and other body organs. Therefore, the HCP would prescribe an increase in the amount of IV lactated Ringer's solution administered per hour. There is nothing in the situation that calls for blood resplacement, which is not used for fluid therapy for burn injuries. Administering a diuretic would not correct the problem because fluid replacement is needed. Diuretics promote the removal of the circulating volume, thereby further compromising the inadequate tissue perfusion. Intravenous 5% dextrose solution is isotonic before administered but is hypotonic once the dextrose is metabolized. Hypotonic solutions are not appropriate for fluid resuscitation of a client with significant burn injuries.

The nurse is assessing a client 24 hours following a cholecystectomy. The nurse notes that the T-tube has drained 750 mL of green-brown drainage since the surgery. Which nursing intervention is most appropriate? 1. Clamp the T-tube. 2. Irrigate the T-tube. 3. Document the findings. 4. Notify the health care provider

3 Following cholecystectomy, drainage from the Ttube is initially bloody and then turns a greenish-brown color. The drainage is measured as output. The amount of expected drainage will range from 500 to 1000 mL/day. The nurse would document the output.

A gastrectomy is performed on a client with gastric cancer. In the immediate postoperative period, the nurse notes bloody drainage from the nasogastric tube. The nurse should take which most appropriate action? 1. Measure abdominal girth. 2. Irrigate the nasogastric tube. 3. Continue to monitor the drainage. 4. Notify the health care provider (HCP)

3 Following gastrectomy, drainage from the nasogastric tube is normally bloody for 24 hours postoperatively, changes to brown-tinged, and is then yellow or clear. Because bloody drainage is expected in the immediate postoperative period, the nurse should continue to monitor the drainage. The nurse does not need to notify the HCP at this time. Measuring abdominal girth is performed to detect the development of distention. Following gastrectomy, a nasogastric tube should not be irrigated unless there are specific HCP prescriptions to do so.

A client has a prescription to take guaifenesin. The nurse determines that the client understands the proper administration of this medication if the client states that he or she will perform which action? 1. Take an extra dose if fever develops 2. Take the medication with meals only 3. Take the tablet with a full glass of water 4. Decrease the amount of daily fluid intake

3 Guaifenesin is an expectorant and should be taken with a full glass of water to decrease the viscosity of secretions. Extra doses should not be taken. The client should contact the health care provider if the cough lasts longer than 1 week or is accompanied by fever, rash, sore throat, or persistent headache. Fluids are needed to decrease the viscosity of secretions. The medication does not have to be taken with meals.

The nurse is analyzing the laboratory results of a client with leukemia who has received a regimen of chemotherapy. Which laboratory value would the nurse specifically note as a result of the massive cell destruction that occurred from the chemotherapy? 1. Anemia 2. Decreased platelets 3. Increased uric acid level 4. Decreased leukocyte count

3 Hyperuricemia is especially common following treatment for leukemias and lymphomas because chemotherapy results in massive cell kill. Although options 1, 2, and 4 also may be noted, an increased uric acid level is related specifically to cell destruction.

The nurse is caring for a client after a bronchoscopy and biopsy. Which finding, if noted in the client, should be reported immediately to the health care provider? 1. Dry cough 2. Hematuria 3. Bronchospasm 4. Blood-streaked sputum

3 If a biopsy was performed during a bronchoscopy, blood-streaked sputum is expected for several hours. Frank blood indicates hemorrhage. A dry cough may be expected. The client should be assessed for signs of complications, which would include cyanosis, dyspnea, stridor, bronchospasm, hemoptysis, hypotension, tachycardia, and dysrhythmias. Hematuria is unrelated to this procedure.

The nurse manager is observing a new nursing graduate caring for burn client in protective isolation. The nurse manager intervenes if the new nursing graduate planned to implement which unsafe component of protective isolation technique? 1. Using sterile sheets and linens 2. Performing strict hand-washing technique 3. Wearing gloves and a gown only when giving direct care to the client 4. Wearing protective garb, including a mask, gloves, cap, shoe covers, gowns, and plastic apron

3 In protective isolation, the nurse needs to protect the client at all times from any potential infectious contact. Thorough hand washing should be done before and after each contact with the burn-injured client. Sterile sheets and linens are used because of the client's high risk for infection. Protective garb, including gloves, cap, masks, shoe covers, gowns, and plastic apron, need to be worn when in the client's room and when directly caring for the client.

The nurse performs a physical assessment on a client with type 2 diabetes mellitus. Findings include a fasting blood glucose level of 120 mg/dL (6.8 mmol/L), temperature of 101 °F (38.3 °C), pulse of 102 beats/minute, respirations of 22 breaths/minute, and blood pressure of 142/72 mm Hg. Which finding would be the priority concern to the nurse? 1. Pulse 2. Respiration 3. Temperature 4. Blood pressure

3 In the client with type 2 diabetes mellitus, an elevated temperature may indicate infection. Infection is a leading cause of hyperosmolar hyperglycemic syndrome in the client with type 2 diabetes mellitus. The other findings are within normal limits.

The nurse should plan to implement which intervention in the care of a client experiencing neutropenia as a result of chemotherapy? 1. Restrict all visitors. 2. Restrict fluid intake. 3. Teach the client and family about the need for hand hygiene. 4. Insert an indwelling urinary catheter to prevent skin breakdown.

3 In the neutropenic client, meticulous hand hygiene education is implemented for the client, family, visitors, and staff. Not all visitors are restricted, but the client is protected from persons with known infections. Fluids should be encouraged. Invasive measures such as an indwelling urinary catheter should be avoided to prevent infections.

A client with tuberculosis is being started on antituberculosis therapy with isoniazid. Before giving the client the first dose, the nurse should ensure which baseline study has been completed? 1. Electrolyte levels 2. Coagulation times 3. Liver enzyme levels 4. Serum creatinine level

3 Isoniazid therapy can cause an elevation of hepatic enzyme levels and hepatitis. Therefore, liver enzyme levels are monitored when therapy is initiated and during the first 3 months of therapy. They may be monitored longer in the client who is older than 50 years or abuses alcohol. The laboratory tests in options 1, 2, and 4 are not necessary.

A client with severe acne is seen in the clinic and the health care provider (HCP) prescribes isotretinoin. The nurse reviews the client's medication record and would contact the HCP if the client is also, taking which medication? 1. Digoxin 2. Phenytoin 3. Vitamin A 4. Furosemide

3 Isotretinoin is a metabolite of vitamin A and can produce a generalized intensification of isotretinoin toxicity. Because of the potential for increased toxicity, vitamin A supplements should be discontinued before isotretinoin therapy. There are no contraindications associated with digoxin, phenytoin, or furosemide.

A client has an as needed prescription for loperamide hydrochloride. For which condition should the nurse administer this medication? 1. Constipation 2. Abdominal pain 3. An episode of diarrhea 4. Hematest-positive nasogastric tube drainage

3 Loperamide is an antidiarrheal agent. It is used to manage acute and chronic diarrhea in conditions such as inflammatory bowel disease. Loperamide also can be used to reduce the volume of drainage from an ileostomy. It is not used for the conditions in options 1, 2, and 4.

The nurse provides instructions to a client who is taking levothyroxine. The nurse should tell the client to take the medication in which way? 1. With food 2. At lunchtime 3. On an empty stomach 4. At bedtime with a snack

3 Oral doses of levothyroxine should be taken on an empty stomach to enhance absorption. Dosing should be done in the morning before breakfast.

A client has begun medication therapy with pancrelipase. The nurse evaluates that the medication is having the optimal intended benefit if which effect is observed? 1. Weight loss 2. Relief of heartburn 3. Reduction of steatorrhea 4. Absence of abdominal pain

3 Pancrelipase is a pancreatic enzyme used in clients with pancreatitis as a digestive aid. The medication should reduce the amount of fatty stools (steatorrhea). Another intended effect could be improved nutritional status. It is not used to treat abdominal pain or heartburn. Its use could result in weight gain but should not result in weight loss if it is aiding in digestion.

The nurse is performing an assessment on a client with pheochromocytoma. Which assessment data would indicate a potential complication associated with this disorder? 1. A urinary output of 50 mL/hour 2. A coagulation time of 5 minutes 3. Aheart rate that is 90 beats/minute and irregular 4. A blood urea nitrogen level of 20 mg/dL (7.1 mmol/L)

3 Pheochromocytoma is a catecholamine-producing tumor usually found in the adrenal medulla, but extraadrenal locations include the chest, bladder, abdomen, and brain; it is typically a benign tumor but can be malignant. Excessive amounts of epinephrine and norepinephrine are secreted. The complications associated with pheochromocytoma include hypertensive retinopathy and nephropathy, myocarditis, increased platelet aggregation, and stroke. Death can occur from shock, stroke, kidney failure, dysrhythmias, or dissecting aortic aneurysm. An irregular heart rate indicates the presence of a dysrhythmia. A coagulation time of 5 minutes is normal. A urinary output of 50 mL/hour is an adequate output. A blood urea nitrogen level of 20 mg/dL (7.1 mmol/L) is a normal finding.

A client has been started on long-term therapy with rifampin. The nurse should provide which information to the client about the medication? 1. Should always be taken with food or antacids 2. Should be double-dosed if 1 dose is forgotten 3. Causes orange discoloration of sweat, tears, urine, and feces 4. May be discontinued independently if symptoms are gone in 3 months

3 Rifampin causes orange-red discoloration of body secretions and will stain soft contact lenses permanently. Rifampin should be taken exactly as directed. Doses should not be doubled or skipped. The client should not stop therapy until directed to do so by a health care provider. It is best to administer the medication on an empty stomach unless it causes gastrointestinal upset, and then it may be taken with food. Antacids, if prescribed, should be taken at least 1 hour before the medication.

Silver sulfadiazine is prescribed for a client with a partial-thickness burn and the nurse provides teaching about the medication. Which statement made by the client indicates a need for further teaching about the treatments? 1. "The medication is an antibacterial." 2. "The medication will help heal the burn." 3. "The medication is likely to cause stinging every time it is applied." 4. "The medication should be applied directly to the wound."

3 Silver sulfadiazine is an antibacterial that has a broad spectrum of activity against gram-negative bacteria, gram-positive bacteria, and yeast. It is applied directly to the wound to assist in healing. It does not cause stinging when applied.

The nurse is monitoring a client for signs and symptoms related to superior vena cava syndrome. Which is an early sign of this oncological emergency? 1. Cyanosis 2. Arm edema 3. Periorbital edema 4. Mental status changes

3 Superior vena cava syndrome occurs when the superior vena cava is compressed or obstructed by tumor growth. Early signs and symptoms generally occur in the morning and include edema of the face, especially around the eyes, and client complaints of tightness of a shirt or blouse collar. As the compression worsens, the client experiences edema of the hands and arms. Cyanosis and mental status changes are late signs.

Tamoxifen citrate is prescribed for a client with metastatic breast carcinoma. The client asks the nurse if her family member with bladder cancer can also take this medication. The nurse most appropriately responds by making which statement? 1. "This medication can be used only to treat breast cancer." 2. "Yes, your family member can take this medication for bladder cancer as well." 3. "This medication can be taken to prevent and treat clients with breast cancer." 4. "This medication can be taken by anyone with cancer as long as their health care provider approves it."

3 Tamoxifen is an antineoplastic medication that competes with estradiol for binding to estrogen in tissues containing high concentrations of receptors. Tamoxifen is used to treat metastatic breast carcinoma in women and men. Tamoxifen is also effective in delaying the recurrence of cancer following mastectomy and for preventing breast cancer in those that are at high risk.

Terbutaline is prescribed for a client with bronchitis. The nurse checks the client's medical history for which disorder in which the medication should be used with caution? 1. Osteoarthritis 2. Hypothyroidism 3. Diabetes mellitus 4. Polycystic disease

3 Terbutaline is a bronchodilator and is contraindicated in clients with hypersensitivity to sympathomimetics. It should be used with caution in clients with impaired cardiac function, diabetes mellitus, hypertension, hyperthyroidism, or a history of seizures. The medication may increase blood glucose levels.

The nurse has taught the client about an upcoming endoscopic retrograde cholangiopancreatography (ERCP) procedure. The nurse determines that the client needs further information if the client makes which statement? 1. "I know I must sign the consent form." 2. "I hope the throat spray keeps me from gagging." 3. "I'm glad I don't have to lie still for this procedure." 4. "I'm glad some intravenous medication will be given to relax me."

3 The client does have to lie still for ERCP, which takes about 1 hour to perform. The client also has to sign a consent form. Intravenous sedation is given to relax the client, and an anesthetic spray is used to help keep the client from gagging as the endoscope is passed.

A client has experienced a pulmonary embolism. The nurse should assess for which symptom, which is most commonly reported? 1. Hot, flushed feeling 2. Sudden chills and fever 3. Chest pain that occurs suddenly 4. Dyspnea when deep breaths are taken

3 The most common initial symptom in pulmonary embolism is chest pain that is sudden in onset. The next most commonly reported symptom is dyspnea, which is accompanied by an increased respiratory rate. Other typical symptoms of pulmonary embolism include apprehension and restlessness, tachycardia, cough, and cyanosis.

The nurse is preparing to administer a dose of naloxone intravenously to a client with an opioid overdose. Which supportive medical equipment should the nurse plan to have at the client's bedside if needed? 1. Nasogastric tube 2. Paracentesis tray 3. Resuscitation equipment 4. Central line insertion tray

3 The nurse administering naloxone for suspected opioid overdose should have resuscitation equipment readily available to support naloxone therapy if it is needed. Other adjuncts that may be needed include oxygen, a mechanical ventilator, and vasopressors.

A client is brought to the emergency department in an unresponsive state, and a diagnosis of hyperosmolar hyperglycemic syndrome is made. The nurse would immediately prepare to initiate which anticipated health care provider's prescription? 1. Endotracheal intubation 2. 100 units of NPH insulin 3. Intravenous infusion of normal saline 4. Intravenous infusion of sodium bicarbonate

3 The primary goal of treatment in hyperosmolar hyperglycemic syndrome (HHS) is to rehydrate the client to restore fluid volume and to correct electrolyte deficiency. Intravenous (IV) fluid replacement is similar to that administered in diabetic ketoacidosis (DKA) and begins with IV infusion of normal saline. Regular insulin, not NPH insulin, would be administered. The use of sodium bicarbonate to correct acidosis is avoided because it can precipitate a further drop in serum potassium levels. Intubation and mechanical ventilation are not required to treat HHS.

The nurse is assessing a client who has a new ureterostomy. Which statement by the client indicates the need for more education about urinary stoma care? 1. "I change my pouch every week." 2. "I change the appliance in the morning." 3. "I empty the urinary collection bag when it is two-thirds full." 4. "When I'm in the shower I direct the flow of water away from my stoma."

3 The urinary collection bag should be changed when it is one-third full to prevent pulling of the appliance and leakage. The remaining options identify correct statements about the care of a urinary stoma.

A client has just been admitted to the nursing unit following thyroidectomy. Which assessment is the priority for this client? 1. Hypoglycemia 2. Level of hoarseness 3. Respiratory distress 4. Edema at the surgical site

3 Thyroidectomy is the removal of the thyroid gland, which is located in the anterior neck. It is very important to monitor airway status, as any swelling to the surgical site could cause respiratory distress. Although all of the options are important for the nurse to monitor, the priority nursing action is to monitor the airway.

Chemotherapy dosage is frequently based on total body surface area (BSA), so it is important for the nurse to perform which assessment before administering chemotherapy? 1. Measure the client's abdominal girth. 2. Calculate the client's body mass index. 3. Measure the client's current weight and height. 4. Ask the client about his or her weight and height.

3 To ensure that the client receives optimal doses of chemotherapy, dosing is usually based on the total BSA, which requires a current accurate height and weight for BSA calculation (before each medication administration). Asking the client about his or her height and weight may lead to inaccuracies in determining a true BSA and dosage. Calculating body mass index and measuring abdominal girth will not provide the data needed.

A client with a peptic ulcer is diagnosed with a Helicobacter pylori infection. The nurse is teaching the client about the medications prescribed, including clarithromycin, esomeprazole, and amoxicillin. Which statement by the client indicates the best understanding of the medication regimen? 1. "My ulcer will heal because these medications will kill the bacteria." 2. "These medications are only taken when I have pain from my ulcer." 3. "The medications will kill the bacteria and stop the acid production." 4. "These medications will coat the ulcer and decrease the acid production in my stomach."

3 Triple therapy for H. pylori infection usually includes 2 antibacterial medications and a proton pump inhibitor. Clarithromycin and amoxicillin are antibacterials. Esomeprazole is a proton pump inhibitor. These medications will kill the bacteria and decrease acid production.

The nurse performs an admission assessment on a client with a diagnosis of tuberculosis. The nurse should check the results of which diagnostic test that will confirm this diagnosis? 1. Chest x-ray 2. Bronchoscopy 3. Sputum culture 4. Tuberculin skin test

3 Tuberculosis is definitively diagnosed through culture and isolation of Mycobacterium tuberculosis. A presumptive diagnosis is made based on a tuberculin skin test, a sputum smear that is positive for acid-fast bacteria, a chest x-ray, and histological evidence of granulomatous disease on biopsy.

A client calls the emergency department and tells the nurse that he came directly into contact with poison ivy shrubs. The client tells the nurse that he cannot see anything on the skin and asks the nurse what to do. The nurse should make which response? 1. "Come to the emergency department." 2. "Apply calamine lotion immediately to the exposed skin areas." 3. "Take a shower immediately, lathering and rinsing several times." 4. "It is not necessary to do anything if you cannot see anything on your skin."

3 When an individual comes in contact with a poison ivy plant, the sap from the plant forms an invisible film on the human skin. The client should be instructed to cleanse the area by showering immediately and to lather the skin several times and rinse each time in running water. Removing the poison ivy sap will decrease the likelihood of irritation. Calamine lotion may be one product recommended for use if dermatitis develops. The client does not need to be seen in the emergency department at this time.

Zafirlukast is prescribed for a client with bronchial asthma. Which laboratory test does the nurse expect to be prescribed before the administration of this medication? 1. Platelet count 2. Neutrophil count 3. Liver function tests 4. Complete blood count

3 Zafirlukast is a leukotriene receptor antagonist used in the prophylaxis and long-term treatment of bronchial asthma. Zafirlukast is used with caution in clients with impaired hepatic function. Liver function laboratory tests should be performed to obtain a baseline, and the levels should be monitored during administration of the medication. It is not necessary to perform the other laboratory tests before administration of the medication.

The nurse is doing an admission assessment on a client with a history of duodenal ulcer. To determine whether the problem is currently active, the nurse should assess the client for which sign(s)/ symptom(s) of duodenal ulcer? 1. Weight loss 2. Nausea and vomiting 3. Pain relieved by food intake 4. Pain radiating down the right arm

3 by food intake. These clients generally describe the pain as a burning, heavy, sharp, or "hungry" pain that often localizes in the mid-epigastric area. The client with duodenal ulcer usually does not experience weight loss or nausea and vomiting. These symptoms are more typical in the client with a gastric ulcer.

The client with hyperparathyroidism is taking alendronate. Which statements by the client indicate understanding of the proper way to take this medication? (SELECT ALL THAT APPLY.) 1. "I should take this medication with food." 2. "I should take this medication at bedtime." 3. "I should sit up for at least 30 minutes after taking this medication." 4. "I should take this medication first thing in the morning on an empty stomach." 5. "I can pick a time to take this medication that best fits my lifestyle as long as I take it at the same time each day."

3, 4 Alendronate is a bisphosphonate used in hyperparathyroidism to inhibit bone loss and normalize serum calcium levels. Esophagitis is an adverse effect of primary concern in clients taking alendronate. For this reason the client is instructed to take alendronate first thing in the morning with a full glass of water on an empty stomach, not to eat or drink anything else for at least 30 minutes after taking the medication, and to remain sitting upright for at least 30 minutes after taking it. A daily dosing schedule and a once-weekly dosing schedule is available for clients taking alendronate.

The nurse is preparing a client with a new diagnosis of hypothyroidism for discharge. The nurse determines that the client understands discharge instructions if the client states which signs and symptoms are associated with this diagnosis? (SELECT ALL THAT APPLY.) 1. Tremors 2. Weight loss 3. Feeling cold 4. Loss of body hair 5. Persistent lethargy 6. Puffiness of the face

3, 4, 5, 6 Feeling cold, hair loss, lethargy, and facial puffiness are signs of hypothyroidism. Tremors and weight loss are signs of hyperthyroidism.

The nurse manager is planning the clinical assignments for the day. Which staff members cannot be assigned to care for a client with herpes zoster? (SELECT ALL THAT APPLY.) 1. The nurse who never had roseola 2. The nurse who never had mumps 3. The nurse who never had chickenpox 4. The nurse who never had German measles 5. The nurse who never received the varicella-zoster vaccine

3, 5 The nurses who have not had chickenpox or did not receive the varicella-zoster vaccine are susceptible to the herpes zoster virus and should not be assigned to care for the client with herpes zoster. Nurses who have not contracted roseola, mumps, or rubella are not necessarily susceptible to herpes zoster. Herpes zoster (shingles) is caused by a reactivation of the varicella-zoster virus, the causative virus of chickenpox. Individuals who have not been exposed to the varicella-zoster virus or who did not receive the varicella-zoster vaccine are susceptible to chickenpox. Health care workers who are unsure of their immune status should have varicella titers done before exposure to a person with herpes zoster.

The nurse instructs a client to use the pursed-lip method of breathing and evaluates the teaching by asking the client about the purpose of this type of breathing. The nurse determines that the client understands if the client states that the primary purpose of pursed-lip breathing is to promote which outcome? 1. Promote oxygen intake 2. Strengthen the diaphragm ' 3. Strengthen the intercostal muscles 4. Promote carbon dioxide elimination

4 Pursed-lip breathing facilitates maximal expiration for clients with obstructive lung disease. This type of breathing allows better expiration by increasing airway pressure that keeps air passages open during exhalation. Options 1, 2, and 3 are not the purposes of this type of breathing.

A client has just had surgery to create an ileostomy. The nurse assesses the client in the immediate postoperative period for which most frequent complication of this type of surgery? 1. Folate deficiency 2. Malabsorption of fat 3. Intestinal obstruction 4. Fluid and electrolyte imbalance

4 A frequent complication that occurs following ileostomy is fluid and electrolyte imbalance. The client requires constant monitoring of intake and output to prevent this from occurring. Losses require replacement by intravenous infusion until the client can tolerate a diet orally. Intestinal obstruction is a less frequent complication. Fat malabsorption and folate deficiency are complications that could occur later in the postoperative period.

A client with small cell lung cancer is being treated with etoposide. The nurse monitors the client during administration, knowing which adverse effect is specifically associated with this medication? 1. Alopecia 2. Chest pain 3. Pulmonary fibrosis 4. Orthostatic hypotension

4 An adverse effect specific to etoposide is orthostatic hypotension. Etoposide should be administered slowly over 30 to 60 minutes to avoid hypotension. The client's blood pressure is monitored during the infusion. Hair loss occurs with nearly all antineoplastic medications. Chest pain and pulmonary fibrosis are unrelated to this medication.

A client with ovarian cancer is being treated with vincristine. The nurse monitors the client, knowing which manifestation indicates an adverse effect specific to this medication? 1. Diarrhea 2. Hair loss 3. Chest pain 4. Peripheral neuropathy

4 An adverse effect specific to vincristine is peripheral neuropathy, which occurs in almost every client. Peripheral neuropathy can be manifested as numbness and tingling in the fingers and toes. Depression of the Achilles tendon reflex may be the first clinical sign indicating peripheral neuropathy. Constipation rather than diarrhea is most likely to occur with this medication, although diarrhea may occur occasionally. Hair loss occurs with nearly all antineoplastic medications. Chest pain is unrelated to this medication.

An external insulin pump is prescribed for a client with diabetes mellitus. When the client asks the nurse about the functioning of the pump, the nurse bases the response on which information about the pump? 1. It is timed to release programmed doses of either short-duration or NPH insulin into the bloodstream at specific intervals. 2. It continuously infuses small amounts of NPH insulin into the bloodstream while regularly monitoring blood glucose levels. 3. It is surgically attached to the pancreas and infuses regular insulin into the pancreas. This releases insulin into the bloodstream. 4. It administers a small continuous dose of short duration insulin subcutaneously. The client can self-administer an additional bolus dose from the pump before each meal.

4 An insulin pump provides a small continuous dose of short-duration (rapid- or short-acting) insulin subcutaneously throughout the day and night. The client can selfadminister an additional bolus dose from the pump before each meal as needed. Short-duration insulin is used in an insulin pump. An external pump is not attached surgically to the pancreas.

A client arriving at the emergency department has experienced frostbite to the right hand. Which finding would the nurse note on assessment of the client's hand? 1. A pink, edematous hand 2. Fiery red skin with edema in the nail beds 3. Black fingertips surrounded by an erythematous rash 4. A white color to the skin, which is insensitive to touch

4 Assessment findings in frostbite include a white or blue color; the skin will be hard, cold, and insensitive to touch. As thawing occurs, flushing of the skin, the development of blisters or blebs, or tissue edema appears. Options 1, 2, and 3 are incorrect.

A client with squamous cell carcinoma of the larynx is receiving bleomycin intravenously. The nurse caring for the client anticipates which diagnostic study will be prescribed? 1. Echocardiography 2. Electrocardiography 3. Cervical radiography 4. Pulmonary function studies

4 Bleomycin is an antineoplastic medication that can cause interstitial pneumonitis, which can progress to pulmonary fibrosis. Pulmonary function studies along with hematological, hepatic, and renal function tests need to be monitored. The nurse needs to monitor lung sounds for dyspnea and crackles, which indicate pulmonary toxicity. The medication needs to be discontinued immediately if pulmonary toxicity occurs. Options 1, 2, and 3 are unrelated to the specific use of this medication.

A client with non-Hodgkin's lymphoma is receiving daunorubicin. Which finding would indicate to the nurse that the client is experiencing an adverse effect related to the medication? 1. Fever 2. Sores in the mouth and throat 3. Complaints of nausea and vomiting 4. Crackles on auscultation of the lungs

4 Cardiotoxicity noted by abnormal electrocardiographic findings or cardiomyopathy manifested as heart failure (lung crackles) is an adverse effect of daunorubicin. Bone marrow depression is also an adverse effect. Fever is a frequent side effect and sores in the mouth and throat can occur occasionally. Nausea and vomiting is a frequent side effect associated with the medication that begins a few hours after administration and lasts 24 to 48 hours. Options 1, 2, and 3 are not adverse effects.

During the admission assessment of a client with advanced ovarian cancer, the nurse recognizes which manifestation is typical of the disease? 1. Diarrhea 2. Hypermenorrhea 3. Abnormal bleeding 4. Abdominal distention

4 Clinical manifestations of ovarian cancer include abdominal distention, urinary frequency and urgency, pleural effusion, malnutrition, pain from pressure caused by the growing tumor and the effects of urinary or bowel obstruction, constipation, ascites with dyspnea, and ultimately general severe pain. Abnormal bleeding, often resulting in hypermenorrhea, is associated with uterine cancer.

A cromolyn sodium inhaler is prescribed for a client with allergic asthma. The nurse provides instructions regarding the adverse effects of this medication and should tell the client that which undesirable effect is associated with this medication? 1. Insomnia 2. Constipation 3. Hypotension 4. Bronchospasm

4 Cromolyn sodium is an inhaled nonsteroidal antiallergy agent and a mast cell stabilizer. Undesirable effects associated with inhalation therapy of cromolyn sodium are bronchospasm, cough, nasal congestion, throat irritation, and wheezing. Clients receiving this medication orally may experience pruritus, nausea, diarrhea, and myalgia.

The nurse is giving discharge instructions to a client with pulmonary sarcoidosis. The nurse concludes that the client understands the information if the client indicates to report which early sign of exacerbation? 1. Fever 2. Fatigue 3. Weight loss 4. Shortness of breath

4 Dry cough and dyspnea are typical early manifestations of pulmonary sarcoidosis. Later manifestations include night sweats, fever, weight loss, and skin nodules.

The nurse provides instructions to a client newly diagnosed with type 1 diabetes mellitus. The nurse recognizes accurate understanding of measures to prevent diabetic ketoacidosis when the client makes which statement? 1. "I will stop taking my insulin if I'm too sick to eat." 2. "I will decrease my insulin dose during times of illness." 3. "I will adjust my insulin dose according to the level of glucose in my urine." 4. "I will notify my health care provider (HCP) if my blood glucose level is higher than 250 mg/dL (14.2 mmol/L)."

4 During illness, the client with type 1 diabetes mellitus is at increased risk of diabetic ketoacidosis, due to hyperglycemia associated with the stress response and due to a typically decreased caloric intake. As part of sick day management, the client with diabetes should monitor blood glucose levels and should notify the HCP if the level is higher than 250 mg/dL(14.2 mmol/ L). Insulin should never be stopped. In fact, insulin may need to be increased during times of illness. Doses should not be adjusted without the HCP's advice and are usually adjusted on the basis of blood glucose levels, not urinary glucose readings.

The nurse has given a client taking ethambutol information about the medication. The nurse determines that the client understands the instructions if the client states that he or she will immediately report which finding? 1. Impaired sense of hearing 2. Gastrointestinal side effects 3. Orange-red discoloration of body secretions 4. Difficulty in discriminating the color red from green

4 Ethambutol causes optic neuritis, which decreases visual acuity and the ability to discriminate between the colors red and green. This poses a potential safety hazard when a client is driving a motor vehicle. The client is taught to report this symptom immediately. The client also is taught to take the medication with food if gastrointestinal upset occurs. Impaired hearing results from antitubercular therapy with streptomycin. Orange-red discoloration of secretions occurs with rifampin.

A client is admitted to the hospital with a suspected diagnosis of Hodgkin's disease. Which assessment finding would the nurse expect to note specifically in the client? 1. Fatigue 2. Weakness 3. Weight gain 4. Enlarged lymph nodes

4 Hodgkin's disease is a chronic progressive neoplastic disorder of lymphoid tissue characterized by the painless enlargement of lymph nodes with progression to extra lymphatic sites, such as the spleen and liver. Weight loss is most likely to be noted. Fatigue and weakness may occur but are not related significantly to the disease.

The nurse manager is teaching the nursing staff about signs and symptoms related to hypercalcemia in a client with metastatic prostate cancer, and tells the staff that which is a late sign or symptom of this oncological emergency? 1. Headache 2. Dysphagia 3. Constipation 4. Electrocardiographic changes

4 Hypercalcemia is a manifestation of bone metastasis in late-stage cancer. Headache and dysphagia are not associated with hypercalcemia. Constipation may occur early in the process. Electrocardiogram changes include shortened ST segment and a widened T wave.

A client arrives at the emergency department following a burn injury that occurred in the basement at home, and an inhalation injury is suspected. What would the nurse anticipate to be prescribed for the client? 1. 100% oxygen via an aerosol mask 2. Oxygen via nasal cannula at 6 L/minute 3. Oxygen via nasal cannula at 15 L/minute 4. 100% oxygen via a tight-fitting, nonrebreather face mask

4 If an inhalation injury is suspected, administration of 100% oxygen via a tight-fitting nonrebreather face mask is prescribed until carboxyhemoglobin levels fall (usually below 15%). In inhalation injuries, the oropharynx is inspected for evidence of erythema, blisters, or ulcerations. The need for endotracheal intubation also is assessed. Administration of oxygen by aerosol mask and cannula are incorrect and would not provide the necessary oxygen supply needed for adequate tissue perfusion for the client with a likely inhalation injury

The nurse is assessing the respiratory status of a client who has suffered a fractured rib. The nurse should expect to note which finding? 1. Slow, deep respirations 2. Rapid, deep respirations 3. Paradoxical respirations 4. Pain, especially with inspiration

4 Rib fractures result from a blunt injury or a fall. Typical signs and symptoms include pain and tenderness localized at the fracture site that is exacerbated by inspiration and palpation, shallow respirations, splinting or guarding the chest protectively to minimize chest movement, and possible bruising at the fracture site. Paradoxical respirations are seen with flail chest.

The evening nurse reviews the nursing documentation in a client's chart and notes that the day nurse has documented that the client has a stage II pressure ulcer in the sacral area. Which finding would the nurse expect to note on assessment of the client's sacral area? 1. Intact skin 2. Full-thickness skin loss 3. Exposed bone, tendon, or muscle 4. Partial-thickness skin loss of the dermis

4 In a stage II pressure ulcer, the skin is not intact. Partial-thickness skin loss of the dermis has occurred. It presents as a shallow open ulcer with a red-pink wound bed, without slough. It may also present as an intact or open/ruptured serum-filled blister. The skin is intact in stage I. Full-thickness skin loss occurs in stage III. Exposed bone, tendon, or muscle is present in stage IV

The nurse is conducting a session about the principles of first aid and is discussing the interventions for snakebite to an extremity. The nurse should inform those attending the session that the first priority intervention in the event of this occurrence is which action? 1. Immobilize the affected extremity. 2. Remove jewelry and constricting clothing from the victim. 3. Place the extremity in a position so that it is below the level of the heart. 4. Move the victim to a safe area away from the snake and encourage the victim to rest.

4 In the event of a snakebite, the first priority is to move the victim to a safe area away from the snake and encourage the victim to rest to decrease venom circulation. Next, jewelry and constricting clothing are removed before swelling occurs. Immobilizing the extremity and maintaining the extremity at the heart level would be done next; these actions limit the spread of the venom. The victim is kept warm and calm. Stimulants such as alcohol or caffeinated beverages are not given to the victim because these products may speed the absorption of the venom. The victim should be transported to an emergency facility as soon as possible.

While giving care to a client with an internal cervical radiation implant, the nurse finds the implant in the bed. The nurse should take which initial action? 1. Call the health care provider (HCP). 2. Reinsert the implant into the vagina. 3. Pick up the implant with gloved hands and flush it down the toilet. 4. Pick up the implant with long-handled forceps and place it in a lead container.

4 In the event that a radiation source becomes dislodged, the nurse would first encourage the client to lie still until the radioactive source has been placed in a safe, closed container. The nurse would use long-handled forceps to place the source in the lead container that should be in the client's room. The nurse should then call the radiation oncologist and document the event and the actions taken. It is not within the scope of nursing practice to insert a radiation implant.

The nurse is caring for a client admitted to the emergency department with diabetic ketoacidosis (DKA). In the acute phase, the nurse plans for which priority intervention? 1. Correct the acidosis. 2. Administer 5% dextrose intravenously. 3. Apply a monitor for an electrocardiogram. 4. Administer short-duration insulin intravenously

4 Lack of insulin (absolute or relative) is the primary cause of DKA. Treatment consists of insulin administration (short- or rapid-acting), intravenous fluid administration(normal saline initially, not 5% dextrose), and potassium replacement, followed by correcting acidosis. Cardiac monitoring is important due to alterations in potassium levels associated with DKA and its treatment, but applying an electrocardiogram monitor is not the priority action.

Megestrol acetate, an antineoplastic medication, is prescribed for a client with metastatic endometrial carcinoma. The nurse reviews the client's history and should contact the health care provider if which diagnosis is documented in the client's history? 1. Gout 2. Asthma 3. Myocardial infarction 4. Venous thromboembolism

4 Megestrol acetate suppresses the release of luteinizing hormone from the anterior pituitary by inhibiting pituitary function and regressing tumor size. Megestrol is used with caution if the client has a history of venous thromboembolism. Options 1, 2, and 3 are not contraindications for this medication.

A client has a new prescription for metoclopramide. On review of the chart, the nurse identifies that this medication can be safely administered with which condition? 1. Intestinal obstruction 2. Peptic ulcer with melena 3. Diverticulitis with perforation 4. Vomiting following cancer chemotherapy

4 Metoclopramide is a gastrointestinal stimulant and antiemetic. Because it is a gastrointestinal stimulant, it is contraindicated with gastrointestinal obstruction, hemorrhage, or perforation. It is used in the treatment of vomiting after surgery, chemotherapy, or radiation.

The nurse has just administered the first dose of omalizumab to a client. Which statement by the client would alert the nurse that the client may be experiencing a life-threatening effect? 1. "I have a severe headache." 2. "My feet are quite swollen." 3. "I am nauseated and may vomit." 4. "My lips and tongue are swollen."

4 Omalizumab is an antiinflammatory used for longterm control of asthma. Anaphylactic reactions can occur with the administration of omalizumab. The nurse administering the medication should monitor for adverse reactions of the medication. Swelling of the lips and tongue are an indication of an anaphylaxis. The client statements in options 1, 2, and 3 are not indicative of an adverse reaction.

A client has an as-needed prescription for ondansetron. For which condition(s) should the nurse administer this medication? 1. Paralytic ileus 2. Incisional pain 3. Urinary retention 4. Nausea and vomiting

4 Ondansetron is an antiemetic used to treat postoperative nausea and vomiting, as well as nausea and vomiting associated with chemotherapy. The other options are incorrect reasons for administering this medication.

The nurse is monitoring a client with a diagnosis of peptic ulcer. Which assessment finding would most likely indicate perforation of the ulcer? 1. Bradycardia 2. Numbness in the legs 3. Nausea and vomiting 4. A rigid, boardlike abdomen

4 Perforation of an ulcer is a surgical emergency and is characterized by sudden, sharp, intolerable severe pain beginning in the mid-epigastric area and spreading over the abdomen, which becomes rigid and board like. Nausea and vomiting may occur. Tachycardia may occur as hypovolemic shock develops. Numbness in the legs is not an associated finding.

The nurse is instructing a hospitalized client with a diagnosis of emphysema about measures that will enhance the effectiveness of breathing during dyspneic periods. Which position should the nurse instruct the client to assume? 1. Sitting up in bed 2. Side-lying in bed 3. Sitting in a recliner chair 4. Sitting up and leaning on an overbed table

4 Positions that will assist the client with emphysema with breathing include sitting up and leaning on an overbed table, sitting up and resting the elbows on the knees, and standing and leaning against the wall.

Silver sulfadiazine is prescribed for a client with a burn injury. Which laboratory finding requires the need for follow-up by the nurse? 1. Glucose level of 99 mg/dL (5.65 mmol/L) 2. Magnesium level of 1.5 mEq/L (0.75 mmol/L) 3. Platelet level of 300,000 mm3 (300 Â 109 /L) 4. White blood cell count of 3000 mm3 (3.0 Â109 /L)

4 Silver sulfadiazine is used for the treatment of burn injuries. Adverse effects of this medication include rash and itching, blue-green or gray skin discoloration, leukopenia, and interstitial nephritis. The nurse should monitor a complete blood count, particularly the white blood cells, frequently for the client taking this medication. If leukopenia develops, the health care provider is notified and the medication is usually discontinued. The white blood cell count noted in option 4 is indicative of leukopenia. The other laboratory values are not specific to this medication, and are also within normal limits.

A client with a gastric ulcer has a prescription for sucralfate 1 gram by mouth 4 times daily. The nurse should schedule the medication for which times? 1. With meals and at bedtime 2. Every 6 hours around the clock 3. One hour after meals and at bedtime 4. One hour before meals and at bedtime

4 Sucralfate is a gastric protectant. The medication should be scheduled for administration 1 hour before meals and at bedtime. The medication is timed to allow it to form a protective coating over the ulcer before food intake stimulates gastric acid production and mechanical irritation. The other options are incorrect.

The camp nurse asks the children preparing to swim in the lake if they have applied sunscreen. The nurse reminds the children that chemical sunscreens are most effective when applied at which times? 1. Immediately before swimming 2. 5 minutes before exposure to the sun 3. Immediately before exposure to the sun 4. At least 30 minutes before exposure to the sun

4 Sunscreens are most effective when applied at least 30 minutes before exposure to the sun so that they can penetrate the skin. All sunscreens should be reapplied after swimming or sweating.

The community health nurse is instructing a group of young female clients about breast self-examination. The nurse should instruct the clients to perform the examination at which time? 1. At the onset of menstruation 2. Every month during ovulation 3. Weekly at the same time of day 4. 1 week after menstruation begins

4 The breast self-examination should be performed regularly, 7 days after the onset of the menstrual period. Performing the examination weekly is not recommended. At the onset of menstruation and during ovulation, hormonal changes occur that may alter breast tissue.

The nurse has conducted discharge teaching with a client diagnosed with tuberculosis who has been receiving medication for 2 weeks. The nurse determines that the client has understood the information if the client makes which statement? 1. "I need to continue medication therapy for 1 month." 2. "I can't shop at the mall for the next 6 months." 3. "I can return to work if a sputum culture comes back negative." 4. "I should not be contagious after 2 to 3 weeks of medication therapy."

4 The client is continued on medication therapy for up to 12 months, depending on the situation. The client generally is considered noncontagious after 2 to 3 weeks of medication therapy. The client is instructed to wear a mask if there will be exposure to crowds until the medication is effective in preventing transmission. The client is allowed to return to work when the results of 3 sputum cultures are negative.

The nurse is assessing a client with multiple trauma who is at risk for developing acute respiratory distress syndrome. The nurse should assess for which earliest sign of acute respiratory distress syndrome? 1. Bilateral wheezing 2. Inspiratory crackles 3. Intercostal retractions 4. Increased respiratory rate

4 The earliest detectable sign of acute respiratory distress syndrome is an increased respiratory rate, which can begin from 1 to 96 hours after the initial insult to the body. This is followed by increasing dyspnea, air hunger, retraction of accessory muscles, and cyanosis. Breath sounds may be clear or consist of fine inspiratory crackles or diffuse coarse crackles.

A client has undergone esophagogastroduodenoscopy. The nurse should place highest priority on which item as part of the client's care plan? 1. Monitoring the temperature 2. Monitoring complaints of heartburn 3. Giving warm gargles for a sore throat 4. Assessing for the return of the gag reflex

4 The nurse places highest priority on assessing for return of the gag reflex. This assessment addresses the client's airway. The nurse also monitors the client's vital signs and for a sudden increase in temperature, which could indicate perforation of the gastrointestinal tract. This complication would be accompanied by other signs as well, such as pain. Monitoring for sore throat and heartburn are also important; however, the client's airway is the priority.

The nurse is caring for a client who sustained superficial partial-thickness burns on the anterior lower legs and anterior thorax. Which finding does the nurse expect to note during the resuscitation/emergent phase of the burn injury? 1. Decreased heart rate 2. Increased urinary output 3. Increased blood pressure 4. Elevated hematocrit levels

4 The resuscitation/emergent phase begins at the time of injury and ends with the restoration of capillary permeability, usually at 48 to 72 hours following the injury. During the resuscitation/emergent phase, the hematocrit level increases to above normal because of hemoconcentration from the large fluid shifts. Hematocrit levels of 50% to 55% (0.50 to 0.55) are expected during the first 24 hours after injury, with return to normal by 36 hours after injury. Initially, blood is shunted away from the kidneys and renal perfusion and glomerular filtration are decreased, resulting in low urine output. The burn client is prone to hypovolemia and the body attempts to compensate by increased pulse rate and lowered blood pressure. Pulse rates are typically higher than normal, and the blood pressure is decreased as a result of the large fluid shifts.

When assessing a lesion diagnosed as basal cell carcinoma, the nurse most likely expects to note which findings? (SELECT ALL THAT APPLY.) 1. An irregularly shaped lesion 2. A small papule with a dry, rough scale 3. A firm, nodular lesion topped with crust 4. A pearly papule with a central crater and a waxy border 5. Location in the bald spot atop the head that is exposed to outdoor sunlight

4, 5 Basal cell carcinoma appears as a pearly papule with a central crater and rolled waxy border. Exposure to ultraviolet sunlight is a major risk factor. A melanoma is an irregularly shaped pigmented papule or plaque with a red-, white-, or blue-toned color. Actinic keratosis, a premalignant lesion, appears as a small macule or papule with a dry, rough, adherent yellow or brown scale. Squamous cell carcinoma is a firm, nodular lesion topped with a crust or a central area of ulceration.

The clinic nurse assesses the skin of a client with psoriasis after the client has used a new topical treatment for 2 months. The nurse identifies which characteristics as improvement in the manifestations of psoriasis? (SELECT ALL THAT APPLY) 1. Presence of striae 2. Palpable radial pulses 3. Absence of any ecchymosis on the extremities 4. Thinner and decrease in the number of reddish papules 5. Scarce amount of silvery-white scaly patches on the arms

4, 5 Psoriasis skin lesions include thick reddened papules or plaques covered by silvery-white patches. A decrease in the severity of these skin lesions is noted as an improvement. The presence of striae (stretch marks), palpable pulses, or lack of ecchymosis is not related to psoriasis.

A client admitted to the hospital with a suspected diagnosis of acute pancreatitis is being assessed by the nurse. Which assessment findings would be consistent with acute pancreatitis? (SELECT ALL THAT APPLY.) 1. Diarrhea 2. Black, tarry stools 3. Hyperactive bowel sounds 4. Gray-blue color at the flank 5. Abdominal guarding and tenderness 6. Left upper quadrant pain with radiation to the back

4, 5, 6 Grayish-blue discoloration at the flank is known as Grey-Turner's sign and occurs as a result of pancreatic enzyme leakage to cutaneous tissue from the peritoneal cavity. The client may demonstrate abdominal guarding and may complain of tenderness with palpation. The pain associated with acute pancreatitis is often sudden in onset and is located in the epigastric region or left upper quadrant with radiation to the back. The other options are incorrect.


Conjuntos de estudio relacionados

Neurology Multiple choice questions

View Set

Ch 1. Introduction to Accounting and Business Test

View Set

Using Automated Medication Dispensing Systems

View Set

Anatomy & Physiology Ch. 1 Pt. 1

View Set

Foundations of Family and Consumer Sciences Education

View Set

Math- Converting different fractions to decimals and percents

View Set